Tag Archives: 2014

Đề thi và đáp án kì thi chọn đội tuyển thi Quốc gia trường Phổ thông Năng khiếu năm học 2013 – 2014

ĐỀ THI

Ngày thi thứ nhất

Bài 1. Tìm tất cả các hàm số $f: \mathbb{R} \rightarrow \mathbb{R}$ thoả mãn:

$\quad\quad\quad\quad\quad\quad\quad\quad\quad\quad f\left(x^3+y+f(y)\right)=2 y+x^2 f(x) \forall x, y \in \mathbb{R}$

Bài 2. Cho dãy số $\left(u_n\right)$ thoả mãn $u_1=2013, u_{n+1}=u_n^3-4 u_n^2+5 u_n \forall n \in \mathbb{N}^*$. Tìm tất cả các số nguyên tố $p$ là ước của $u_{2014}+2009$ và $p \equiv 3(\bmod 4)$.

Bài 3. Trong một hội nghị khoa học có 5000 đại biểu tham dự, trong đó có một nhóm gồm 280 người là ban tổ chức. Giả sử rằng mỗi thành viên của ban tổ chức thì quen tất cả thành viên khác của hội nghị. Một uỷ ban gồm một số đại biểu được gọi là uỷ ban làm việc nếu tất cả các thành viên trong đó đều quen nhau, và được gọi là uỷ ban thách thức nếu không có hai thành viên nào của uỷ ban quen nhau. Chứng minh rằng ta có thể phân hoạch các đại biểu (kể cả ban tổ chức) thành các ủy ban mà mỗi ủy ban gồm 5 đại biểu, sao cho các uỷ ban này hoặc là làm việc, hoặc là thách thức.

Bài 4. Cho tam giác $A B C$ có $B, C$ cố định còn $A$ di động sao cho $A B=A C$ và $\angle B A C>60^{\circ}$. Đường thẳng đối xứng với $B C$ qua $A B$ cắt $A C$ tại $P$. Trên đoạn $P C$ lấy điểm $M$ sao cho $P M=P B$. Gọi $N$ là giao điểm của $A B$ và phân giác ngoài của góc $\angle B C A$. Chứng minh rằng $M N$ luôn đi qua một điểm cố định.

Ngày thi thứ hai

Bài 5. Cho 2014 số thực $x_1, x_2, \ldots, x_{2014}$ thoả mãn các điều kiện:

(i) $x_1+x_2+\cdots+x_{2014}=0$,

(ii) $x_1^2+x_2^2+\cdots+x_{2014}^2=2014$.

Tìm giá trị lớn nhất của biểu thức $P=x_1 x_2 \cdots x_{2014}$.

Bài 6. Cho dãy số $\left(u_n\right)$ xác định bởi $u_1=1, u_{n+1}=\frac{u_n}{\sqrt{u_n^2+1}+\sqrt{2}}, \forall n \in \mathbb{N}^*$. Tính $\lim \frac{u_{n+1}}{u_n}$

Bài 7. Cho $n$ là số nguyên dương và $A$ là tập con khác rỗng của $X={1,2, \ldots, n}$.

(a) Tính giá trị của tổng $S(A)=\sum_{E \subset X}(-1)^{|E \cap A|}$ trong đó $E$ lấy trên tất cả các tập hợp con của $X$, kể cả tập hợp rỗng.

(b) Cho $m \in \mathbb{N}^*$. Xét $m$ tập con khác rỗng của $X$ là $A_1, A_2, \ldots, A_m$ và $m$ số nguyên khác 0 là $a_1, a_2, \ldots, a_m$ sao cho $a_1+a_2+\cdots+a_m<0$. Chứng minh rằng tồn tại $E \subset X$ sao cho $\sum_{i=1}^m(-1)^{\left|E \cap A_i\right|} a_i>0$.

Bài 8. Cho tam giác $A B C$ nhọn có $H$ là trực tâm. $P$ là điểm di động bên trong tam giác $A B C$ sao cho $\angle B P C=\angle B H C$. Đường thẳng qua $B$ vuông góc $A B$ cắt $P C$ tại $M$, đường thẳng qua $C$ vuông góc $A C$ cắt $P B$ tại $N$. Gọi $I$ là trung điểm của $M N$. Chứng minh rằng $I$ luôn thuộc một đường thẳng cố định.

 

LỜI GIẢI

Ngày thi thứ nhất

Bài 1 . Tìm tất cả các hàm số $f: \mathbb{R} \rightarrow \mathbb{R}$ thoả mãn:

$\quad\quad\quad\quad\quad\quad f\left(x^3+y+f(y)\right)=2 y+x^2 f(x), \forall x, y \in \mathbb{R} .$

Lời giải. Trong phương trình đã cho, thay $x=y=0$, ta có $f(f(0))=0$.

Lại thay $y=0$ thì

$\quad\quad\quad\quad\quad\quad\quad\quad\quad\quad f\left(f^3+f(0)\right)=x^2 f(x), \forall x .$

Thay $y=f(0)$ thì

$\quad\quad\quad\quad\quad\quad\quad\quad\quad\quad f\left(x^3+f(0)\right)=2 f(0)+x^2 f(x) .$

Từ đây suy ra $f(0)=0$. Thay $y=0$ vào đẳng thức đã cho ta được $f\left(x^3\right)=x^2 f(x)$. Do đó ta có

$\quad\quad\quad f\left(x^3+y+f(y)\right)=2 y+f\left(x^3\right) \text { hay } f(x+y+f(y))=2 y+f(x)\quad\quad\quad(*).$

Thay $y$ bởi $-y$, ta được

$\quad\quad\quad\quad\quad\quad\quad\quad\quad\quad f(x-y+f(-y))=-2 y+f(x) .$

Với $x$ bất kì, ta lấy $2 y=f(x)$ ta được $f(x-y+f(-y))=0$ suy ra $x-y+f(-y)=0$. Do đó, ta được $f(-x)=f(-y+f(-y))=-2 y=-f(x)$. Từ đây suy ra

$\quad\quad\quad\quad\quad\quad\quad\quad\quad\quad f(x+f(y)+f(f(y)))=2 f(y)+f(x)$

$\quad\quad\quad\quad\quad\quad\quad\quad\quad\quad f(x+f(y)+f(f(y)))=2 f(y)+f(x) .$

Trong $(*)$ thay $x=-y$ ta được $f(f(y))=2 y+f(-y)=2 y-f(y)$, kết hợp với đẳng thức trên, ta được

$f(x+2 y)=2 f(y)+f(x) .$

Đến đây cho $x=0$ ta được $f(2 y)=2 f(y)$ nên ta được $f(x+y)=f(x)+f(y)$, tức là $f(x)$ cộng tính. Đến đây ta sẽ tính $f\left((x+1)^3+(x-1)^3\right)$ theo hai cách như sau

  • $f\left((x+1)^3+(x-1)^3\right)=f\left(2 x^3+6 x\right)=2 x^2 f(x)+6 f(x)$.
  • $f\left((x+1)^3+(x-1)^3\right)=(x+1)^2 f(x+1)+(x-1)^2 f(x-1)=(x+1)^2(f(x)+$ $f(1))+(x-1)^2(f(x)-f(1))=2 x^2 f(x)+2 f(x)+4 x f(1)$.

So sánh hai đẳng thức trên, ta được $f(x)=x f(1)=a x$ với mọi $x$. Thử lại ta được $a=1, a=-2$.

Vậy các hàm cần tìm là $f(x)=x, f(x)=-2 x$.

Bài 2. Cho dãy số $\left(u_n\right)$ thoả mãn $u_1=2013, u_{n+1}=u_n^3-4 u_n^2+5 u_n \forall n \in \mathbb{N}^*$.

Tìm tất cả các số nguyên tố $p$ là ước của $u_{2014}+2009$ và $p \equiv 3(\bmod 4)$.

Lời giải. Ta có

$\quad\quad\quad\quad\quad\quad\quad\quad u_{n+1}-2 =\left(u_n-2\right)\left(u_{n-1}-1\right)^2 $

$\quad\quad\quad\quad\quad\quad\quad\quad\quad\quad\quad\quad =\left(u_{n-2}-1\right)^2\left(u_{n-1}-1\right)^2\left(u_{n-2}-2\right) $

$\quad\quad\quad\quad\quad\quad\quad\quad\quad\quad\quad\quad =\left(u_{n-1}-1\right)^2\left(u_{n-2}-1\right)^2 \cdots\left(u_2-1\right)^2\left(u_1-2\right) .$

Do đó

$\quad\quad\quad\quad\quad u_{2014}+2009=2011\left[\left(u_{2013}-1\right)^2\left(u_{2012}-1\right)^2 \cdots\left(u_2-1\right)^2+1\right] .$

Gọi $B$ là biểu thức trong dấu ngoặc vuông thứ hai. Ta có bổ đề quen thuộc là nếu $a^2+b^2$ chia hết cho số nguyên tố $p=4 k+3$ thì $a, b$ cùng chia hết cho $p$. Từ đây suy ra số $B$ có dạng $a^2+1$ nên nó sẽ không có ước nguyên tố dạng $4 k+3$.

Vậy $u_{2014}+9$ chỉ có một ước nguyên tố $p \equiv 3(\bmod 4)$ duy nhất là 2011 .

Bài 3. Trong một hội nghị khoa học có 5000 đại biểu tham dự, trong đó có một nhóm gồm 280 người là ban tổ chức. Giả sử rằng mỗi thành viên của ban tổ chức thì quen tất cả thành viên khác của hội nghị. Một uỷ ban gồm một số đại biểu được gọi là uỷ ban làm việc nếu tất cả các thành viên trong đó đều quen nhau, và được gọi là uỷ ban thách thức nếu không có hai thành viên nào của uỷ ban quen nhau. Chứng minh rằng ta có thể phân hoạch các đại biểu (kể cả ban tổ chức) thành các ủy ban mà mỗi ủy ban gồm 5 đại biểu, sao cho các uỷ ban này hoặc là làm việc, hoặc là thách thức.

Lời giải. Trước hết, ta chứng minh bổ đề sau

ĐỊNH LÝ RAMSEY Với $s, t$ là các số nguyên dương, gọi $R(s, t)$ là số đỉnh ít nhất cần có của một graph để trong đó luôn tồn tại một tập độc lập $s$ đỉnh hoặc một graph con đầy đủ $t$ đỉnh. Khi đó

$\quad\quad\quad\quad\quad\quad\quad\quad\quad\quad\quad\quad R(s, t) \leq C_{s+t-2}^{s-1}\quad\quad\quad(*) .$

Chứng minh. Ta sẽ chứng minh rằng

$\quad\quad\quad\quad\quad\quad\quad\quad R(s, t) \leq R(s-1, t)+R(s, t-1) .$

Để ý rằng với $s=1$ hoặc $t=1$ thì $R(s, t)=1$. Do đó, nếu chứng minh được đánh giá này thì chỉ cần dùng tính chất của tam giác Pascal để có

$\quad\quad\quad\quad\quad\quad\quad\quad R(s, t) \leq C_{s+t-3}^{s-2}+C_{s+t-3}^{s-1}=C_{s+t-2}^{s-1} .$

Đặt $n$ là vế phải của $(*)$ và xét graph $G$ có $n$ đỉnh. Xét $v \in G$ thì

  1. Nếu như có ít nhất $R(s, t-1)$ đỉnh kề với $v$. Khi đó, theo định nghĩa thì trong tập đỉnh đó, sẽ luôn có một tập độc lập $s$ đỉnh hoặc một graph con đầy đủ $t-1$ đỉnh, ghép thêm đỉnh $v$ vào thì thỏa mãn điều kiện của $R(s, t)$.
  2. Nếu như có ít nhất $R(s-1, t)$ đỉnh không kề với $v$. Tương tự trên, trong tập đỉnh đó, cũng sẽ có một một graph con đầy đủ $t$ đỉnh hoặc tập độc lập $s-1$ đỉnh, ghép thêm đỉnh $v$ vào thì thỏa mãn điều kiện của $R(s, t)$.

Từ đó, ta thấy graph $G$ này thỏa mãn điều kiện của $R(s, t)$ nên theo tính nhỏ nhất thì $R(s, t) \leq n$.

Trở lại bài toán,

Xét graph đơn vô hướng $G=(V, E)$ đại diện cho hội nghị khoa học đã nêu, trong đó $V$ là tập hợp các đại biểu và hai đỉnh được nối nhau nếu hai đại biểu tương ứng quen nhau. Ta gọi $T$ là tập hợp đỉnh biểu diễn cho thành viên của ban tổ chức.

Khi đó một ủy ban gồm 5 thành viên là đại diện nếu như đó là một graph đầy đủ, còn đó là thách thức nếu đó là graph không có cạnh. Ta gọi các graph con như thế là graph con “chuẩn”.

Trong các đỉnh $V \backslash T$, ta xóa dần dần các graph con chuẩn đến khi không thực hiện được nữa. Ta gọi tập hợp còn lại là $S$. Ta sẽ chứng minh rằng $S \cup T$ có thể phân hoạch thành các graph con chuẩn như trên.

Theo định lý Ramsey, rõ ràng $|S| \leq C_8^4=70$. Xét một đỉnh $v \in S$ thì giả thiết, $v$ kề với cả 280 đỉnh của $T$ nên ta chọn ra trong đó 4 đỉnh để ghép với $v$ tạo thành một graph con “chuẩn”. Cứ như thế thực hiện cho đến hết các phần tử của $S$, còn lại bao nhiêu phần tử trong $T$ thì chia đều ra thành các graph con “chuẩn” là được.

Bài toán được giải quyết.

Nhận xét. Bài toán gốc của đề thi bị sai và có thể chỉ ra phản ví dụ. Nội dung cụ thể của nó như sau: Trong một hội nghi khoa học có 5000 đại biểu tham dư, mỗi một đại biểu biết ít nhất một thứ tiếng. Một uỷ ban gồm một số đại biểu được gọi là uỷ ban làm việc nếu tất cả các thành viên uỷ ban đều biết chung một thứ tiếng, và được gọi là uỷ ban thách thức nếu không có hai thành viên nào của uỷ ban biết chung một thứ tiếng. Uỷ ban có thể gồm 1 thành viên, khi đó gọi là uỷ ban làm việc hay thách thức cũng được. Chứng minh rằng ta có thể chia các đai biều thành đúng 100 uỷ ban, mỗi đại biểu thuộc đúng 1 uỷ ban, sao cho các uỷ ban này hoặc là uỷ ban làm việc, hoặc là uỷ ban thách thúc.

Ban biên tập đã bổ sung giả thiết như bài toán vừa giải ở trên và cố gắng giữ nguyên các phát biểu còn lại.

Bài 4. Cho tam giác $A B C$ có $B, C$ cố định còn $A$ di động sao cho $A B=A C$ và $\angle B A C>60^{\circ}$. Đường thẳng đối xứng với $B C$ qua $A B$ cắt $A C$ tại $P$. Trên đoạn $P C$ lấy điểm $M$ sao cho $P M=P B$. Gọi $N$ là giao điểm của $A B$ và phân giác ngoài của góc $\angle B C A$. Chứng minh rằng $M N$ luôn đi qua một điểm cố định.

Lời giải. Tam giác $P B M$ cân tại $P$ nên bằng biến đổi góc, ta có

$\quad\quad\quad\quad\quad \angle P B M=\angle P M B \Rightarrow 2 \angle A B C-\angle M B C=\angle A C B+\angle M B C .$

Do đó $\angle A B C=2 \angle M B C$ nên $B M$ là tia phân giác của $\angle A B C$. Theo tính chất đường phân giác thì

$\quad\quad\quad\quad\quad\quad\quad\quad\quad\quad\quad\quad \frac{M C}{M A}=\frac{B C}{B A}=\frac{B C}{A C}$

 

Lại có $C N$ là phân giác ngoài của $\angle A C B$ nên ta cũng có $\frac{N A}{N B}=\frac{C A}{C B}$. Gọi $I$ là trung điểm của $B C$ thì $I$ là điểm cố định.

Xét tam giác $A B C$ với $I$ thuộc $B C, M$ thuộc $A C$ và $N$ thuộc $A B$ với

$\quad\quad\quad\quad\quad\quad\quad\quad\quad\quad\quad\quad \frac{I B}{I C} \cdot \frac{M C}{M A} \cdot \frac{N A}{N B}=1 \cdot \frac{B C}{A C} \cdot \frac{A C}{B C}=1$

thì theo định lý Menelaus đảo, ta có $M, N, I$ thẳng hàng.

Vậy $M N$ luôn đi qua điểm $I$ cố định.

Ngày thi thứ hai

Bài 5. Cho 2014 số thực $x_1, x_2, \ldots, x_{2014}$ thoả mãn các điều kiện

$\quad (i) x_1+x_2+\cdots+x_{2014}=0$,

$\quad (ii) x_1^2+x_2^2+\cdots+x_{2014}^2=2014$.

Tìm giá trị lớn nhất của biểu thức $P=x_1 x_2 \cdots x_{2014}$.

Lời giải. Rõ ràng có thể chọn giá trị các biến thích hợp để $P>0$ nên để tìm giá trị lớn nhất của $P$ thì ta chỉ xét các số $x_1, x_2, \ldots, x_{2014}$ đều khác 0 và số các số âm là chẵn. Không mất tính tổng quát, giả sử $x_1 \geq x_2 \geq \ldots \geq x_{2 m}>0>x_{2 m+1} \geq$ $\ldots \geq x_{2014}$. Đổi dấu các số $y_k=-x_k>0$ với $2 m+1 \leq k \leq 2014$. Khi đó ta viết lại

$\quad\quad\quad\quad\quad\quad \left\{\begin{array}{l}x_1+x_2+\cdots+x_{2 m}=y_1+y_2+\cdots+y_{2 n}=A \\ x_1^2+x_2^2+\cdots+x_{2 m}^2+y_1^2+y_2^2+\cdots+y_{2 n}^2=2014\end{array}\right.$

trong đó $m+n=1007$ (ngoài ra $m, n>0$ vì các số đã cho không thể toàn bộ là dương hoặc toàn bộ là âm). Theo bất đẳng thức Cauchy-Schwarz thì

$\quad\quad\quad\quad\quad\quad\quad\quad\quad\quad\quad\quad 2014 \geq \frac{A^2}{2 m}+\frac{A^2}{2 n} \text { nên } A^2 \leq 4 m n$

Lại theo bất đẳng thức AM-GM thì

$\quad\quad\quad\quad\quad\quad\quad\quad\quad P =\left(x_1 x_2 \ldots x_{2 m}\right)\left(y_1 y_2 \ldots y_{2 n}\right) \leq\left(\frac{A}{2 m}\right)^{2 m}\left(\frac{A}{2 n}\right)^{2 n} $

$\quad\quad\quad\quad\quad\quad\quad\quad\quad\quad =\frac{A^{2 m+2 n}}{2^{2 m+2 n} m^{2 m} n^{2 n}} \leq \frac{(4 m n)^{m+n}}{2^{2 m+2 n} m^{2 m} n^{2 n}}=\left(\frac{m}{n}\right)^{n-m}$

Do $m, n$ khác tính chẵn lẻ nên với vai trò bình đẳng của $m, n$, ta có thể giả sử $m<n$ nên $n-m \geq 1$ và $m \leq 503$. Khi đó, áp dụng bất đẳng thức Bernoulli thì

$\quad\quad\quad \left(\frac{n}{m}\right)^{n-m} \geq 1+\left(\frac{n}{m}-1\right)(n-m)=1+\frac{(n-m)^2}{m} \geq 1+\frac{1}{503}=\frac{504}{503}.$

Suy ra $P \leq\left(\frac{m}{n}\right)^{n-m} \leq \frac{503}{504}$. Đây chính là giá trị lớn nhất cần tìm, dấu bằng xảy ra khi $m=503, n=504$ và

$\quad\quad\quad x_1=x_2=\cdots=x_{1006}=\sqrt{\frac{504}{503}}, x_{1007}=x_{1008}=\cdots=x_{2014}=-\sqrt{\frac{503}{504}} .$

Bài 6. Cho dãy số $\left(u_n\right)$ xác định bởi $u_1=1, u_{n+1}=\frac{u_n}{\sqrt{u_n^2+1}+\sqrt{2}} \forall n \in \mathbb{N}^*$.

$\operatorname{Tính} \lim \frac{u_{n+1}}{u_n}$.

Lời giải. Xét hàm số $f(x)=\frac{x}{\sqrt{x^2+1}+\sqrt{2}}$ với $x \in \mathbb{R}$ thì

$\quad\quad\quad\quad\quad\quad\quad\quad f^{\prime}(x)=\frac{1+\sqrt{2+2 x^2}}{\sqrt{1+x^2}\left(\sqrt{2}+\sqrt{1+x^2}\right)^2}>0$

nên hàm này đồng biến trên $\mathbb{R}$. Dãy số đã cho được viết lại thành

$\quad\quad\quad\quad\quad\quad\quad\quad\quad\quad \left\{\begin{array}{l}u_1=1, \\ u_{n+1}=f\left(u_n\right), n \geq 1\end{array}\right.$

thì $u_1<u_2$ nên dễ dàng chứng minh quy nạp được rằng dãy này giảm.

Do dãy này bị chặn dưới bởi 0 nên nó có giới hạn, đặt giới hạn đó là $L \geq 0$. Trong biểu thức xác định dãy, cho $n \rightarrow+\infty$, ta được

$\quad\quad\quad\quad\quad\quad\quad\quad\quad\quad L=\frac{L}{\sqrt{L^2+1}+\sqrt{2}}$

nên $L=0$. Từ đó suy ra

Bài 7 . Cho $n$ là số nguyên dương và $A$ là tập con khác rỗng của $X={1,2, . ., n}$.

(a) Tính giá trị của tổng $S(A)=\sum_{E \subset X}(-1)^{|E \cap A|}$ trong đó $E$ lấy trên tất cả các tập hợp con của $X$, kể cả tập hợp rỗng.

(b) Cho $m \in \mathbb{N}^*$. Xét $m$ tập con khác rỗng của $X$ là $A_1, A_2, \ldots, A_m$ và $m$ số nguyên khác 0 là $a_1, a_2, \ldots, a_m$ sao cho $a_1+a_2+\cdots+a_m<0$. Chứng minh rằng tồn tại $E \subset X$ sao cho $\sum_{i=1}^m(-1)^{\left|E \cap A_i\right|} a_i>0$.

Lời giải. (a) Nếu $A=X$ thì

$\quad\quad\quad\quad\quad\quad\quad\quad\quad\quad\quad S(A)=\sum_{E \subset X}(-1)^{|E|}=C_n^0-C_n^1+C_n^2-\cdots+(-1)^n C_n^n=0 .$

Còn nếu $A \neq X$, do $S(A)$ chỉ phụ thuộc vào số phần tử của $A$ nên không mất tính tổng quát, ta giả sử rằng $A={1,2, \ldots, k}$ với $k<n$. Khi đó, ta có

$\quad\quad\quad\quad\quad\quad\quad S(A) =\sum_{E \subset X-{k}}(-1)^{|E \cap A|}+\sum_{E \subset X-{k}}(-1)^{|(E \cup{k}) \cap A|} $

$\quad\quad\quad\quad\quad\quad\quad\quad\quad\quad =\sum_{E \subset X-{k}}(-1)^{|E \cap A|}-\sum_{E \subset X-{k}}(-1)^{|E \cap A|}=0 .$

Vậy $S(A)=0, \forall A \subset X$.

(b) Đặt $f(E)=\sum_{i=1}^m(-1)^{\left|E \cap A_i\right|} a_i$. Giả sử $f(E) \leq 0, \forall E$. Mà ta cũng có

$\quad\quad\quad\quad\quad\quad\quad\quad\quad\quad \sum_{E \subset X} f(E)=\sum_{i=1}^m a_i S\left(A_i\right)=0$

Suy ra $f(E)=0, \forall E \subset X$, nhưng điều này là không thể vì $f(\varnothing)<0$. Vậy luôn tồn tại $E$ sao cho $f(E)>0$.

Bài 8. Cho tam giác $A B C$ nhọn có $H$ là trực tâm. $P$ là điểm di động bên trong tam giác $A B C$ sao cho $\angle B P C=\angle B H C$. Đường thẳng qua $B$ vuông góc $A B$ cắt $P C$ tại $M$, đường thẳng qua $C$ vuông góc $A C$ cắt $P B$ tại $N$. Gọi $I$ là trung điểm của $M N$. Chứng minh rằng $I$ luôn thuộc một đường thẳng cố định.

Lời giải. Vẽ đường kính $A A^{\prime}$ của đường tròn $(A B C)$. Vì $A^{\prime} B \perp A B$ nên $B, A^{\prime}, M$ thẳng hàng. Tương tự thì $C, A^{\prime}, N$ thẳng hàng. Giả sử $B A^{\prime}, C A^{\prime}$ cắt lại $(B H C)$ lần lượt tại $E, F$. Mặt khác

$\quad\quad\quad\angle N P M=180^{\circ}-\angle B H C=\angle A=180^{\circ}-\angle B A^{\prime} C=\angle M A^{\prime} N$

nên $P A^{\prime} M N$ là tứ giác nội tiếp.

Ta sẽ chứng minh trung điểm của $A^{\prime} F, A^{\prime} E, M N$ là thẳng hàng. Theo định lý Menelaus đảo thì điều nào tương đương với

$\quad\quad\quad\quad \frac{\overline{A^{\prime} F}}{\overline{A^{\prime} N}}=\frac{\overline{E A^{\prime}}}{\overline{E M}} \Leftrightarrow \frac{\overline{A^{\prime} F}}{\overline{\overline{A^{\prime} E}}}=-\frac{\overline{A^{\prime} N}}{\overline{E M}} \Leftrightarrow \frac{A^{\prime} B}{A^{\prime} C}=\frac{A^{\prime} N}{M E}\quad\quad(*) .$

Vì $\angle B N A^{\prime}=\angle C M E$ và $\angle N B A^{\prime}=\angle M C E$ nên hai tam giác $B N A^{\prime}, C M E$ dồng dạng với nhau. Do đó

$\quad\quad\quad\quad\quad\quad\quad\quad\quad\quad \frac{A^{\prime} N}{M E}=\frac{A^{\prime} B}{C E} .$

Mặt khác, bằng biến đổi góc, ta cũng có tam giác $C A^{\prime} E$ cân tại $C$ nên $C E=C A^{\prime}$. Ta có được

$\quad\quad\quad\quad\quad\quad\quad\quad\quad\quad \frac{A^{\prime} N}{M E}=\frac{A^{\prime} B}{A^{\prime} C} .$

Do đó, khẳng định $(*)$ là đúng. Vậy nên điểm $I$ luôn nằm trên đường trung bình của tam giác $A^{\prime} E F$ là đường cố định.

 

 

 

 

 

 

 

 

 

 

 

 

 

 

 

 

 

 

 

 

 

 

 

 

 

 

 

 

 

 

 

 

 

 

 

 

 

 

 

 

 

 

 

 

 

 

 

 

 

 

 

ĐỀ THI VÀO LỚP 10 CHUYÊN TOÁN TP.HCM – NĂM 2014

Bài 1. (a) Giải phương trình: $x \sqrt{2 x-3}=3 x-4$

(b) Cho 3 số thực $x, y, z$ thỏa mãn điều kiện: $x+y+z=0 ; x y z \neq 0$. Tính giá trị biểu thức:

$P=\frac{x^{2}}{y^{2}+z^{2}-x^{2}}+\frac{y^{2}}{z^{2}+x^{2}-y^{2}}+\frac{z^{2}}{x^{2}+y^{2}-z^{2}}$

Bài 2. Giải hệ phương trình: $\left\{\begin{array}{l}x+y+\frac{1}{y}=\frac{9}{x} \\ x+y-\frac{4}{x}=\frac{4 y}{x^{2}}\end{array}\right.$

Bài 3. Cho tam giác đều $A B C$ và $M$ là một điểm bất kì trên cạnh $B C$. Gọi $D, E$ lần lượt là hình chiếu vuông góc của $M$ trên $A B$ và $A C$. Xác định vị trí của $M$ để tam giác $M D E$ có chu vi nhỏ nhất.

Bài 4. (a) Cho $x, y$ là 2 số thực khác 0 . Chứng minh rằng: $\frac{x^{2}}{y^{2}}+\frac{y^{2}}{x^{2}} \geq \frac{x}{y}+\frac{y}{x}$

(b) Cho $a, b$ là hai số dương. Tìm giá trị nhỏ nhất của biểu thức: $P=\frac{a^{2}+3 a b+b^{2}}{\sqrt{a b}(a+b)}$

Bài 5. Từ một điểm $M$ nằm ngoài đường tròn $(\mathrm{O})$, kẻ các tiếp tuyến $M A, M B$ với $(\mathrm{O})$ $(A, B$ là các tiếp điểm $)$. Gọi $H$ là giao điểm của $A B$ với $O M, I$ là trung điểm của $M H$. Đường thẳng $A I$ cắt $(\mathrm{O})$ tại điểm $K(K$ khác $A)$.

(a) Chứng minh $H K$ vuông góc với $A I$.

(b) Tính số đo góc $\angle M K B$.

Bài 6. Tìm cặp số nguyên $(x, y)$ thỏa mãn phương trình:

$2015\left(x^{2}+y^{2}\right)-2014(2 x y+1)=25$

LỜI GIẢI

 

Bài 1.

a) Giải phương trình: $x \sqrt{2 x-3}=3 x-4$

b) Cho 3 số thực $x, y, z$ thỏa mãn điều kiện: $x+y+z=0 ; x y z \neq 0$. Tính giá trị biểu thức:

$P=\frac{x^{2}}{y^{2}+z^{2}-x^{2}}+\frac{y^{2}}{z^{2}+x^{2}-y^{2}}+\frac{z^{2}}{x^{2}+y^{2}-z^{2}}$

Lời giải.

a) Giải phương trình: $x \sqrt{2 x-3}=3 x-4 Đ \mathrm{~K} Đ: x \geq \frac{3}{2}$

Phương trình đã cho tương đương với:

$x^{2}(2 x-3)=9 x^{2}-24 x+16 \Leftrightarrow 2 x^{3}-12 x^{2}+24 x-16=0 $

$\Leftrightarrow x^{3}-6 x^{2}+12 x-8=0 \Leftrightarrow(x-2)^{3}=0 \Leftrightarrow x=2$

Ta thấy $x=2$ thỏa yêu cầu bài toán, vậy $x=2$ là nghiệm duy nhất của phương trình.

b) Cho 3 số thực $x, y, z$ thỏa mãn điều kiện: $x+y+z=0 ; x y z \neq 0$. Tính giá trị biểu thức:

$P=\frac{x^{2}}{y^{2}+z^{2}-x^{2}}+\frac{y^{2}}{z^{2}+x^{2}-y^{2}}+\frac{z^{2}}{x^{2}+y^{2}-z^{2}}$

Ta có:

$y+z=-x \Leftrightarrow y^{2}+2 y z+z^{2}=x^{2} \Leftrightarrow y^{2}+z^{2}-x^{2}=-2 y z $

$x+z=-y \Leftrightarrow x^{2}+2 x z+z^{2}=y^{2} \Leftrightarrow x^{2}+z^{2}-y^{2}=-2 x z $

$y+x=-z \Leftrightarrow y^{2}+2 y x+x^{2}=z^{2} \Leftrightarrow y^{2}+x^{2}-z^{2}=-2 y x$

Từ đó ta tính được $P$ :

$P=\frac{x^{2}}{-2 y z}+\frac{y^{2}}{-2 x z}+\frac{z^{2}}{-2 y x}=\frac{x^{3}+y^{3}+z^{3}}{-2 x y z}$

Chú ý:

$x^{3}+y^{3}+z^{3}-3 x y z=0 \Rightarrow x^{3}+y^{3}+z^{3}=3 x y z$

Vậy: $P=\frac{x^{3}+y^{3}+z^{3}}{-2 x y z}=\frac{3 x y z}{-2 x y z}=\frac{-3}{2}$

Bài 2. Giải hệ phương trình: $\left\{\begin{array}{l}x+y+\frac{1}{y}=\frac{9}{x} \\ x+y-\frac{4}{x}=\frac{4 y}{x^{2}}\end{array}\right.$

Lời giải. ĐKXĐ: $x, y \neq 0$

Lấy phương trình (1) trừ phương trình (2) ta thu được:

$\frac{1}{y}+\frac{4}{x}=\frac{9}{x}-\frac{4 y}{x^{2}}  \Leftrightarrow \frac{1}{y}=\frac{5}{x}-\frac{4 y}{x^{2}} \Leftrightarrow x^{2}=5 x y-4 y^{2} \Leftrightarrow x^{2}-5 x y+4 y^{2}=0 $

$\Leftrightarrow(x-4 y)(x-y)=0 \Leftrightarrow\left[\begin{array}{l}x=4 y \\ x=y\end{array}\right.$

Trường hợp 1: $x=4 y$. Thay vào phương trình (1) ta có:

$5 y+\frac{1}{y}=\frac{9}{4 y} \Leftrightarrow 5 y=\frac{5}{4 y} \Leftrightarrow\left[\begin{array} { l }{ y = \frac { 1 } { 2 } } \\ { y = \frac { – 1 } { 2 } }\end{array} \Leftrightarrow \left[\begin{array}{l}x=2, y=\frac{1}{2} \\ x=-2, y=\frac{-1}{2}\end{array}\right.\right.$

Trường hợp $2: x=y$. Thay vào phương trình (1) ta có:

$2 y+\frac{1}{y}=\frac{9}{y} \Leftrightarrow 2 y=\frac{8}{y} \Leftrightarrow\left[\begin{array}{l}y=2 \\ y=-2\end{array} \Leftrightarrow\left[\begin{array}{l}x=2, y=2 \\ x=-2, y=-2\end{array}\right.\right.$

Vậy tập nghiệm của phương trình là: $(x, y)=(2,2),(-2,-2),\left(2, \frac{1}{2}\right),\left(-2, \frac{-1}{2}\right)$

Bài 3. Cho tam giác đều $A B C$ và $M$ là một điểm bất kì trên cạnh $B C$. Gọi $D, E$ lần lượt là hình chiếu vuông góc của $M$ trên $A B$ và $A C$. Xác định vị trí của $M$ để tam giác MDE có chu vi nhỏ nhất.

Lời giải.

  • Gọi độ dài cạnh tam giác đều là $a$.

Ta có $M D \cdot A B+M E \cdot A C=2 S_{A M D}+2 S_{A M C}=2 S_{A B C}$. Hay $(M D+M E)=A H \cdot a$, suy ra $M D+M E=A H$ không đổi.

  • Ta có $D, E$ thuộc đường tròn đường kính $A M$. Vẽ đường kính $D F$, ta có $\angle D F E=$ $\angle D A E=60^{\circ}$.

Suy ra $D E=D F \sin D F E=A M \sin 60^{\circ}$.

$D E$ nhỏ nhất khi và chỉ khi $A M$ nhỏ nhất, khi và chỉ khi $M$ trùng với $H$ trung điểm $B C$.

  • Vậy chu vi tam giác $M D E$ nhỏ nhất khi và chỉ khi $M$ là trung điểm $B C$.

Bài 4.

a) Cho $x, y$ là 2 số thực khác 0 . Chứng minh rằng: $\frac{x^{2}}{y^{2}}+\frac{y^{2}}{x^{2}} \geq \frac{x}{y}+\frac{y}{x}$

b) Cho $a, b$ là hai số dương. Tìm giá trị nhỏ nhất của biểu thức:

$P=\frac{a^{2}+3 a b+b^{2}}{\sqrt{a b}(a+b)}$

Lời giải.

a) Bằng biến đổi tương đương ta có:

$\frac{x^{2}}{y^{2}}+\frac{y^{2}}{x^{2}}-\left(\frac{x}{y}+\frac{y}{x}\right) \geq 0 \Leftrightarrow \frac{(x-y)^{2}\left(\left(x+\frac{1}{2}\right)^{2}+\frac{3}{4} y^{2}\right)}{x^{2} y^{2}} \geq 0$

Bất đẳng thức cuối luôn đúng. Dấu bằng trong bất đẳng thức xảy ra khi $x=y$.

b) Cách 1: Với $a, b$ là hai số dương. Ta có:

$P=\frac{a^{2}+3 a b+b^{2}}{\sqrt{a b}(a+b)}=\frac{(a+b)^{2}+a b}{\sqrt{a b}(a+b)}=\frac{\frac{1}{4}(a+b)^{2}+a b+\frac{3}{4}(a+b)^{2}}{\sqrt{a b}(a+b)} $

$P=\frac{\frac{1}{4}(a+b)^{2}+a b}{\sqrt{a b}(a+b)}+\frac{\frac{3}{4}(a+b)}{\sqrt{a b}}$

Áp dụng bất đẳng thức Cauchy:

$P=\frac{\frac{1}{4}(a+b)^{2}+a b}{\sqrt{a b}(a+b)}+\frac{\frac{3}{4}(a+b)}{\sqrt{a b}} \geq \frac{2 \sqrt{\frac{1}{4} a b(a+b)^{2}}}{\sqrt{a b}(a+b)}+\frac{\frac{3}{4} \cdot 2 \sqrt{a b}}{\sqrt{a b}}=1+\frac{3}{2}=\frac{5}{2}$

Dấu bằng trong bất đẳng thức xảy ra khi $a=b$

Cách 2: Ta có:

$P=\frac{a^{2}+3 a b+b^{2}}{\sqrt{a b}(a+b)}=\frac{(a+b)^{2}+a b}{\sqrt{a b}(a+b)}=\frac{a+b}{\sqrt{a b}}+\frac{\sqrt{a b}}{a+b}=\frac{3}{4} \cdot \frac{a+b}{\sqrt{a b}}+\frac{1}{4} \cdot \frac{a+b}{\sqrt{a b}}+\frac{\sqrt{a b}}{a+b}$

Áp dụng bất đẳng thức Cauchy:

$P \geq \frac{3}{4} \cdot 2+2 \sqrt{\frac{1}{4} \cdot \frac{a+b}{\sqrt{a b}} \cdot \frac{\sqrt{a b}}{a+b}}=\frac{3}{2}+1=\frac{5}{2}$

Dấu bằng trong bất đẳng thức xảy ra khi $a=b$

Bài 5.Từ một điểm $M$ nằm ngoài đường tròn $(\mathrm{O})$, kẻ các tiếp tuyến $M A, M B$ với $(\mathrm{O})$ $(A, B$ là các tiếp điểm $)$. Gọi $H$ là giao điểm của $A B$ với $O M, I$ là trung điểm của $M H$. Đường thẳng $A I$ cắt $(\mathrm{O})$ tại điểm $K(K$ khác $A)$.

a) Chứng minh $H K$ vuông góc với $A I$.

b) Tính số đo góc $\angle M K B$.

Lời giải.

a) Vẽ đường kính $A C, C H$ cắt $A I$ tại $K^{\prime}$.

Dễ thấy hai tam giác $A B C$ và $M H A$ đồng dạng, từ đó suy ra $A C H$ và $M A I$ đồng dạng.

Suy ra $\angle A C H=\angle M A I$, mà $\angle M A I+\angle I A C=90^{\circ}$, suy ra $\angle A C H+\angle I A C=$ $90^{\circ}$.

Do đó $\angle A K^{\prime} C=90^{\circ}$, suy ra $K^{\prime}$ thuộc $(O)$, từ đó $K^{\prime} \equiv K$. Ta có điều cần chứng minh.

b) Ta có $I K \cdot I A=I H^{2}=I M^{2}$.

Suy ra $\triangle I K M \backsim \triangle I M A$, do đó $\angle I M K=\angle I A M=\angle K B H$.

Từ đó tứ giác $B H K M$ nội tiếp, suy ra $\angle B K M=\angle B H M=90^{\circ}$.

Bài 6. Tìm cặp số nguyên $(x, y)$ thỏa mãn phương trình:

$2015\left(x^{2}+y^{2}\right)-2014(2 x y+1)=25$

Lời giải.

Ta có: $2015\left(x^{2}+y^{2}\right)-2014(2 x y+1)=25$

$\Leftrightarrow 2014(x-y)^{2}+x^{2}+y^{2}=2039$

Vậy: $2014(x-y)^{2} \leq 2039 \Leftrightarrow|x-y| \leq 1$

  • Trường hợp 1: $x-y=0$. Ta có: $x^{2}+y^{2}=2039$

Phương trình này không có nghiệm nguyên vì 2039 không chia hết cho $2 .$

  • Trường hợp 2: $x-y=1$. Ta có: $y^{2}+y-12=0$

Phương trình này có nghiệm $y=3$ hay $y=-4$

Từ đó ta có hai cặp nghiệm của phương trình là: $(x, y)={(4 ; 3),(-3 ;-4)}$

  • Trường hợp 3: $x-y=-1$. Ta có: $y^{2}-y-12=0$

Phương trình này có nghiệm $y=-3$ hay $y=4$

Từ đó ta có hai cặp nghiệm của phương trình là: $(x, y)={(-4 ;-3),(3 ; 4)}$

Vậy tập nghiệm của phương trình là: $(x, y)={(4 ; 3),(-3 ;-4),(3 ; 4),(-4 ;-3)}$

 

 

 

 

 

 

 

 

 

 

 

 

 

 

 

 

 

 

 

 

 

 

 

 

 

 

 

 

 

ĐỀ THI VÀO LỚP 10 CHUYÊN TOÁN TRƯỜNG PHỔ THÔNG NĂNG KHIẾU 2014

Bài 1. Cho phương trình $\left(m^{2}+5\right) x^{2}-2 m x-6 m=0(1)$ với $m$ là tham số.

(a) Tìm $m$ sao cho phương trình (1) có hai nghiệm phân biệt. Chứng minh rằng khi đó tổng của hai nghiệm không thể là số nguyên.

(b) Tìm $m$ sao cho phương trình (1) có hai nghiệm $x_{1}, x_{2}$ thỏa mãn điều kiện

$\left(x_{1} x_{2}-\sqrt{x_{1}+x_{2}}\right)^{4}=16$

Bài 2. (a) Giải hệ phương trình $\left\{\begin{array}{l}2(1+x \sqrt{y})^{2}=9 y \sqrt{x} \\ 2(1+y \sqrt{x})^{2}=9 x \sqrt{y}\end{array}\right.$

(b) Cho tam giác $A B C$ vuông tại $A$ với các đường phân giác trong $B M, C N$.

Chứng minh bất đẳng thức $\frac{(M C+M A)(N B+N A)}{M A \cdot N A} \geq 3+2 \sqrt{2}$.

Bài 3. Cho các số nguyên dương $a, b$ thỏa $\frac{1}{a}+\frac{1}{b}=\frac{1}{c}$.

(a) Chứng minh rằng $a+b$ không thể là số nguyên tố.

(b) Chứng minh rằng nếu $c>1$ thì $a+c$ và $b+c$ không thể đồng thời là số nguyên tố.

Bài 4. Cho điểm $C$ thay đổi trên nửa đường tròn đường kính $A B=2 R(C \neq A, C \neq$ $B)$. Gọi $H$ là hình chiếu vuông góc của $C$ lên $A B ; I$ và $J$ lần lượt là tâm đường tròn nội tiếp các tam giác $A C H$ và $B C H$. Các đường thẳng $C I, C J$ cắt $A B$ tại $M, N$.

(a) Chứng minh $A N=A C, B M=B C$.

(b) Chứng minh 4 điểm $M, N, I, J$ cùng nằm trên một đường tròn và các đường thẳng $M J, N I$ và $C H$ đồng quy.

(c) Tìm giá trị lớn nhất của $\mathrm{MN}$ và giá trị lớn nhất của diện tích tam giác $\mathrm{CMN}$ theo $R$.

Bài 5. Cho 5 số tự nhiên phân biệt sao cho tổng của ba số bất kỳ trong chúng lớn hơn tổng của hai số còn lại.

(a) Chứng minh rằng tất cả 5 số đã cho đều không nhỏ hơn 5 .

(b) Tìm tất cả các bộ gồm 5 số thỏa mãn đề bài mà tổng của chúng nhỏ hơn $40 .$

LỜI GIẢI

 

Bài 1. Cho phương trình $\left(m^{2}+5\right) x^{2}-2 m x-6 m=0(1)$ với $m$ là tham số.

(a) Tìm $m$ sao cho phương trình (1) có hai nghiệm phân biệt. Chứng minh rằng khi đó tổng của hai nghiệm không thể là số nguyên.

(b) Tìm $m$ sao cho phương trình (1) có hai nghiệm $x_{1}, x_{2}$ thỏa mãn điều kiện

$\left(x_{1} x_{2}-\sqrt{x_{1}+x_{2}}\right)^{4}=16$

Lời giải.

a) Phương trình có hai nghiêm phân biệt khi và chỉ khi:

$\left\{\begin{array}{l}m^{2}+5 \neq 0 \\\Delta^{\prime}=m^{2}+6 m\left(m^{2}+5\right)>0\end{array}\right.$

$\Leftrightarrow m\left(6 m^{2}+m+30\right)>0$

$\Leftrightarrow m\left[5 m^{2}+\left(m+\frac{1}{2}\right)+\frac{119}{4}\right]>0$

$\Leftrightarrow m>0$

Khi đó theo định lý Viete ta có $x_{1}+x_{2}=\frac{2 m}{m^{2}+5}$.

Vi $m^{2}+5-2 m=(m-1)^{2}+4>0$, suy ra $m^{2}+5>2 m>0$.

Do đó $0<\frac{2 m}{m^{2}+5}<1$ nên tổng hai nghiệm của phương trình không thể là số nguyên.

b) Điều kiện để phương trình có hai nghiệm $\Delta^{\prime} \geq 0 \Leftrightarrow m \geq 0 .$ Khi đó $\left\{\begin{array}{l}x_{1}+x_{2}=\frac{2 m}{m^{2}+5} \\ x_{1} x_{2}=\frac{-6 m}{m^{2}+5}\end{array}\right.$

Ta có $\left(x_{1} x_{2}-\sqrt{x_{1}+x_{2}}\right)^{4}=16 \Leftrightarrow\left[\begin{array}{l}x_{1} x_{2}-\sqrt{x_{1}+x_{2}}=2 \\ x_{1} x_{2}-\sqrt{x_{1}+x_{2}}=-2\end{array}\right.$

Trường hợp 1: $x_{1} x_{2}-\sqrt{x_{1}+x_{2}}=2 \Leftrightarrow \frac{-6 m}{m^{2}+5}-\sqrt{\frac{2 m}{m^{2}+5}}=2$.

Đặt $t=\sqrt{\frac{2 m}{m^{2}+5}}$, ta có phương trình: $-3 t^{2}-t=2(V N)$

Trường hợp 2: $x_{1} x_{2}-\sqrt{x_{1}+x_{2}}=-2 \Leftrightarrow \frac{-6 m}{m^{2}+5}-\sqrt{\frac{2 m}{m^{2}+5}}=-2$.

Đặt $t=\sqrt{\frac{2 m}{m^{2}+5}}$ ta có phương trình: $-3 t^{2}-t=-2 \Leftrightarrow t=-1(l), t=\frac{3}{2}$.

Với $t=\frac{3}{2}$ ta có $\frac{2 m}{m^{2}+5}=\frac{3}{2}$. Giải ra được $m=2(n), m=\frac{5}{2}(n)$.

Bài 2. a) Giải hệ phương trình $\left\{\begin{array}{l}2(1+x \sqrt{y})^{2}=9 y \sqrt{x} \\ 2(1+y \sqrt{x})^{2}=9 x \sqrt{y}\end{array}\right.$

b) Cho tam giác $A B C$ vuông tại $A$ với các đường phân giác trong $B M, C N$. Chứng minh bât đẳng thức $\frac{(M C+M A)(N B+N A)}{M A \cdot N A} \geq 3+2 \sqrt{2}$.

Lời giải.

1) Đặt $a=x \sqrt{y}, b=y \sqrt{x}$. Điều kiện $a, b \geq 0$.

Ta có hệ: $\left\{\begin{array}{l}2(1+a)^{2}=9 b(1) \\ 2(1+b)^{2}=9 a(2)\end{array}\right.$

Lấy (1) trừ (2) ta có: $(a-b)(2 a+2 b+13)=0 \Leftrightarrow\left[\begin{array}{l}a=b(n) \\ 2 a+2 b=-13(l)\end{array}\right.$

Với $a=b$ thế vào $(1)$ ta có $2\left(1+a^{2}\right)=9 a \Leftrightarrow\left[\begin{array}{l}a=2, b=2 \\ a=\frac{1}{2}, b=\frac{1}{2}\end{array}\right.$

Khi $a=b=2$ ta có $x=y=\sqrt[3]{4}$

Khi $a=b=\frac{1}{2}$ ta có $x=y=\sqrt[3]{\frac{1}{2}}$.

2) Áp dụng tính chất đường phân giác ta có:

$\frac{M C}{M A}=\frac{B C}{A B}$,  suy ra  $\frac{M C+M A}{M A}=1+\frac{B C}{A B} $

$\frac{B B}{N A}=\frac{B N+N A}{A C}$,  suy ra  $\frac{B N+N A}{N A}=1+\frac{B C}{A C} $

Suy ra:

$\frac{(M C+M A)(N B+N A)}{M A \cdot N A}=\left(1+\frac{B C}{A B}\right)\left(1+\frac{B C}{A C}\right) $

$=1+\frac{B C^{2}}{A B \cdot A C}+\frac{B C}{A B}+\frac{B C}{A C} $

Ta có  $B C^{2}=A B^{2}+A C^{2} \geq 2 \cdot A B \cdot A C$, suy ra $\frac{B C C^{2}}{A B \cdot A C} \geq 2 $

Và  $\frac{B A}{A C}+\frac{B C}{A C} \geq \sqrt{\frac{B C \cdot B C}{A B \cdot A C}} \geq 2 \sqrt{2} . $

Do đó  $\frac{(M C+M A)(N B+N A)}{M A \cdot N A} \geq 3+2 \sqrt{2} .$

Bài 3. Cho các số nguyên dương $a, b$ thỏa $\frac{1}{a}+\frac{1}{b}=\frac{1}{c}$.

a) Chứng minh rằng $a+b$ không thể là số nguyên tố.

b) Chứng minh rằng nếu $c>1$ thì $a+c$ và $b+c$ không thể đồng thời là số nguyên tố.

Lời giải.

a) Từ đề bài ta có $c(a+b)=a b$, suy ra $a b$ chia hết cho $a+b$.

Giả sử $a+b$ nguyên tố. Ta có $a<a+b$, suy ra $a, a+b$ nguyên tố cùng nhau, suy ra $b$ chia hết cho $a+b$ vô lý vì $b<a+b$.

b) Giả sử $a+c, b+c$ đều là các số nguyên tố. Khi đó:

$c(a+b)=a b \Leftrightarrow c a=a b-b c \Leftrightarrow a(b+c)=b(2 a-c) . $

Và  $b(a+c)=a(2 b-c) .$

Dễ thấy $b+c$ nguyên tố và $b+c>b$ nên $b+c$ và $b$ là nguyên tố cùng nhau; tương tự $a+b$ và $a$ nguyên tố cùng nhau.

Mà $a(b+a)$ chia hết cho $b$, suy ra $a$ chia hết cho $b, b(a+c)$ chia hết cho $a$, suy ra $b$ chia hết cho $a$. Suy ra $a=b=2 c$, suy ra $a+c=b+c=3 c$ không phải là số nguyên tố do $c>1$.

Vậy khi $c>1$ thì $a+c, b+c$ không thể đồng thời là số nguyên tố.

Bài 4. Cho điểm $C$ thay đổi trên nửa đường tròn đường kính $A B=2 R(C \neq A, C \neq B)$. Gọi $H$ là hình chiếu vuông góc của $C$ lên $A B ; I$ và $J$ lần lượt là tâm đường tròn nội tiếp các tam giác $A C H$ và $B C H$. Các đường thẳng $C I, C J$ cắt $A B$ tại $M, N$.

a) Chứng minh $A N=A C, B M=B C$.

b) Chứng minh 4 điểm $M, N, I, J$ cùng nằm trên một đường tròn và các đường thẳng $M J, N I$ và $C H$ đồng quy.

c) Tìm giá trị lớn nhất của $\mathrm{MN}$ và giá trị lớn nhất của diện tích tam giác $\mathrm{CMN}$ theo $R$.

 

Lời giải.

a) Ta có $\angle H C B=\angle C A B$ (cùng phụ với $\angle A B C$ ) và $\angle H C A=\angle C B A$ (cùng phụ với $\angle B A C$ ).

Ta có $\angle C A N=\angle N A C+\angle A B C=\angle H A N+\angle A C B=\angle C A N$. Suy ra tam giác $C A N$ cân tại $A$ hay $A N=A C$. Chứng minh tương tự ta có $B M=B C$.

b) Tam giác $C A N$ cân tại $A$ có $A I$ là phân giác nên cũng là trung trực, suy ra $I C=$ $I N$, suy ra $\angle I N C=\angle I C N=\angle I C H+\angle N C H=\frac{1}{2} \angle A C H+\frac{1}{2} \angle B C H=45^{\circ} .$ Tương tự thì $\angle J M C=45^{\circ}$.

Tứ giác $M I J N$ có $\angle J M C=\angle I N C=45^{\circ}$ nên là tứ giác nội tiếp, hay $M, N, I, J$ cùng thuộc một đường tròn.

Tam giác $I N C$ cân có $\angle I C N=45^{\circ}$ nên $\angle C I N=90^{\circ}$, suy ra $C I \perp C M$.

Chứng minh tương tự $M J \perp C N$.

Tam giác $C M N$ có $C H, M J, N I$ là các đường cao nên đồng quy.

c) Đặt $A C=b, B C=a$. Ta có $a^{2}+b^{2}=B C^{2}=4 R^{2}$.

Ta có $A N=A C=b, B M=B C=a$.

$A M+B N=B C+M N$, suy ra $M N=a+b-B C=a+b-2 R$.

Ta có $(a+b)^{2} \leq 2\left(a^{2}+b^{2}\right)=8 R^{2}$. Suy ra $a+b \leq 2 \sqrt{2} R$, suy ra $a+b-2 R \leq$ $2 R(\sqrt{2}-1)$.

Đẳng thức xảy ra khi $a=b=R \sqrt{2}$.

Vậy giá trị lớn nhất của $M N$ bằng $2 R(\sqrt{2}-1)$ khi $C$ là điểm chính giữa đường tròn. Khi đó $S_{C M N}=\frac{1}{2} C H . M N \leq R^{2}(\sqrt{2}-1)$. Đẳng thức xảy ra khi $C$ là điểm chính giữa đường tròn.

Bài 5. Cho 5 số tự nhiên phân biệt sao cho tổng của ba số bất kỳ trong chúng lớn hơn tổng của hai số còn lại.

a) Chứng minh rằng tất cả 5 số đã cho đều không nhỏ hơn 5 .

b) Tìm tất cả các bộ gồm 5 số thỏa mãn đề bài mà tổng của chúng nhỏ hơn 40 .

Lời giải.

a) Gọi 5 số đó là $a, b, c, d, e$, do các số là phân biệt nên ta có thể giả sử $a<b<c<$ $d<e$.

Theo giả thiết ta có $a+b+c>d+e$, suy ra $a+b+c \geq d+e+1$. Suy ra $a \geq d+e+1-b-c$.

Mặt khác, do $b, c, d, e$ là số tự nhiên nên từ $d>c>b$ ta có $d \geq c+1 \geq b+2$, suy ra $d-b \geq 2$.

$e>d>c$, suy ra $e-c \geq 2$.

Do đó $a \geq(d-b)+(e-c)+1 \geq 5$. Suy ra $b, c, d, e>5$.

Vậy các số đều không nhỏ hơn 5 .

b) Nếu $a \geq 6$, suy ra $b \geq 7, c \geq 8, d \geq 9$, e $\geq 10$, suy ra $a+b+c+d+e \geq 40$ ( vô lý), suy ra $a<6$. Theo câu a ta có $a=5$. Khi đó $b+c+5 \geq d+e+1$, suy ra $b+c \geq d+e-4 .$

Mà $d-2 \geq b, e-2 \geq c$, suy ra $d+e-4 \geq b+c$. Do đó $b=d-2, c=e-2$. Khi đó $a+b+c+d+e=5+2 b+2 c+4<40$. Suy ra $b+c<\frac{31}{2}$. Suy ra $b \geq 7$.

Từ đó ta có $b=6, b=7$.

Nếu $b=6$ ta có $d=8, c=8, e=10$. Ta có bộ $(5,6,7,8,9)$

Nếu $b=7, d=9, c=8, e=10$. Ta có bộ $(5,7,8,9,10)$.

 

 

 

 

 

 

 

 

 

 

 

 

 

 

 

 

 

 

 

 

Đề thi học kì 1 lớp 10 chuyên toán trường PTNK năm 2014

Bài 1. Cho hàm số $y=x|x-4|$
a) Vẽ đồ thị $(\mathrm{C})$ của hàm số.
b) Cho đường thẳng $(\mathrm{d}): y=m x$ ( $\mathrm{m}$ là tham số). Tìm $\mathrm{m}$ để $(\mathrm{d})$ cắt $(\underline{\mathrm{C}})$ tại $\mathrm{A}, \mathrm{B}$ khác gốc tọa độ và $A B=2 \sqrt{2}$.

Bài 2. Giải các phương trình và hệ phương trình sau:
a) $2+\sqrt{4 x^{2}-10 x+7}=2 x+\sqrt{3-2 x} \quad$ b) $\left\{\begin{array}{l}x+\dfrac{1}{x^{2}+1}=y+\dfrac{1}{y^{2}+1} \\ \sqrt{y^{2}+\dfrac{4}{x^{2}}}=\dfrac{x^{2}+x-2}{y}\end{array}\right.$
Bài 3 .
a) Cho số tự nhiên $\mathrm{n}$ thỏa $C_{n}^{2}+C_{n+1}^{3}+2 n=128$. Tìm số hạng không chứa $x$ trong khai triển $P(x)=\left(\sqrt{x}-\frac{2}{3 \sqrt[4]{x}}\right)^{n+1},(x>0)$.
b) Cho các số tự nhiên $\mathrm{m}, \mathrm{n}, \mathrm{k}$ thỏa $0 \leq m \leq k \leq n$. Chứng minh rằng $C_{n}^{k} C_{k}^{m}=C_{n}^{m} C_{n-m}^{k-m}$

Bài 4. Lớp 10 Toán có 6 bạn học sinh nữ và 30 bạn học sinh nam.
a) Cần chọn ra 10 bạn để tham gia kéo co trong đó có 5 bạn nam và 5 bạn nữ. Hỏi có bao nhiêu cách chọn?
b) Cần chọn ra 5 bạn để thể hiện một tiết mục văn nghệ, hỏi có bao nhiêu cách chọn có it nhất 2 bạn nam và î nhất 1 bạn mữ?

Bài 5. Cho tam giác đều $\mathrm{ABC}$ nội tiếp đường tròn $(\mathrm{O})$ bán kính $\mathrm{R}$. $\mathrm{AO}$ cắt $(\mathrm{O})$ tại $\mathrm{D}$.
a) Chứng minh rằng với mọi điểm $\mathrm{M}$ thì $\overrightarrow{M B} \cdot \overrightarrow{M C}=\overrightarrow{M D} \cdot \overrightarrow{M O}-\frac{R^{2}}{2}$
b) Cho $\mathrm{M}$ thay đổi trên $(\mathrm{O})$. Tìm giá trị lớn nhất của $\overrightarrow{M B} \cdot \overrightarrow{M C}-\overrightarrow{M D} \cdot \overrightarrow{M A}$
c) Cho điểm $M$ thay đổi trên cạnh $A B, D M$ cắt $(O)$ tại $N$. Xác định $M$ để phương tích của
$\mathrm{D}$ với đường tròn ngoại tiếp tam giác $\mathrm{AMN}$ bằng $2 \mathrm{R}^{2}$.
d) Cho điểm $M$ thay đổi trên đoạn $A D$. ( $K$ ) là đường tròn qua $M$ và tiếp xúc với $(O)$ tại $B .$
Đường tròn $(\mathrm{K})$ cắt đường tròn đường kính $\mathrm{AM}$ tại $\mathrm{T}$. Chứng minh đường thẳng $\mathrm{MT}$ đi qua một điểm cố định $\mathrm{E}$. Tính phương tích của $\mathrm{E}$ đối với $(\mathrm{O})$.

Đáp án thi chọn Đội Tuyển Trường PTNK năm học 2013-2014

Đề thi và đáp án kì thi chọn đội tuyển Toán trường Phổ thông Năng khiếu – ĐHQG TPHCM được tổ chức vào tháng 10 năm 2013, chọn ra 6 học sinh dự thi kì thi HSG Quốc gia năm 2014. Các thí sinh từ các lớp 11, 12 (chủ yếu là học sinh chuyên toán), thực hiện bài thi trong 2 ngày, mỗi ngày 4 bài, mỗi bài 180 phút. Sau đây là đề thi và đáp án thực hiện bởi Star Education.

Ngày thi thứ 1

Bài 1. Tìm tất cả các hàm số $f: \mathbb{R} \rightarrow \mathbb{R}$ thoả mãn

$$f(x^{3}+y+f(y))=2 y+x^{2} f(x), \forall x, y \in \mathbb{R}$$

Bài 2. Cho dãy $\left\{u_{n}\right\}$ thoả mãn $u_{1}=2013, u_{n+1}=u_{n}^{3}-4 u_{n}^{2}+5 u_{n} \forall n \in \mathbb{N}^{*}$. Tìm tất cả các số nguyên tố $p$ là ước của $\left(u_{2014}+2009\right)$ và $p \equiv 3(\bmod 4)$.

Bài 3. Trong một hội nghị khoa học có 5000 đại biểu tham dự, mỗi một đại biểu biết ít nhất một thứ tiếng. Một uỷ ban gồm một số đại biểu được gọi là “uỷ ban làm viẹc” nếu tất cả thành viên trong uỷ ban đều biết chung một thú tiếng; gọi là “uỷ ban thách thức” nếu không có hai thành viên nào của uỷ ban biết chung một thứ tiếng (uỷ ban có thểgồm 1 thành viên; uỷ ban này gọi là làm việc họ̆c thách thức đều được). Chứng minh rằng có thể chia các đại biểu thành 100 uỷ ban rời nhau (mỗi đại biểu thuộc một uỷ ban) sao cho các uỷ ban này họ̆c là uỷ ban làm việc hoặc là uỷ ban thách thức.

Bài 4. Tam giác $A B C$ có $B, C$ cố định còn $A$ di động sao cho $A B=A C$ và $\angle B A C>60^{\circ} .$ Đường thẳng đối xúng với $B C$ qua $A B$ cắt AC tai $P$. Trên đoạn $P C$ lấy $M$ sao cho $P M=P B$. Gọi $N$ là giao điểm của $A B$ với phân giác ngoài góc BCA. Chứng minh $M N$ luôn đi qua một điểm cố định.

Ngày thi thứ 2

Bài 5. Cho 2014 số thực $x_{1}, x_{2}, \ldots, x_{2014}$ thỏa mãn điều kiện $\sum_{i=1}^{2014} x_{i}=0$ và $\sum_{i=1}^{2014} x_{i}^{2}=2014$. Tìm giá trị lớn nhất của biểu thức $P=x_{1} x_{2} \cdots x_{2014}$.

Bài 6. Cho dãy số $u_{n}$ xác định bởi $u_{1}=1, u_{n+1}=\frac{u_{n}}{\sqrt{u_{n}^{2}+1}+\sqrt{2}}$ với mọi $n \in \mathbb{N}^{*}$. Tìm giới hạn $\lim \frac{u_{n+1}}{u_{n}}$.

Bài 7. Cho n nguyên dương và A là tập con khác rỗng của $X={1,2, \ldots, n}$.

  1. Tính giá trị của tổng $S(A)=\sum_{E C X} \cdot(-1)^{|E \cap A|}$,trong đó $E$ lấy trên tất cả các tập con của tập $X$ (kể cả tập rỗng).

  2. Cho $m \in \mathbb{N}^{*}$,xét $m$ tập con khác rỗng của $X$ là $A_{1}, A_{2}, \ldots, A_{m}$ và $m$ số nguyên khác không là $a_{1}, a_{2}, \ldots, a_{m}$ sao cho $a_{1}+a_{2}+\cdots+a_{m}<0$. Chứng minh tồn tại tập con $E$ của $X$ sao cho $\sum_{i=1}^{m}(-1)^{|E \cap A|} a_{i}>0$ (Kí hiệu $|A|$ chỉ số phần tử của tập $A$, số phần tử của tập rỗng là 0 ).

Bài 8. Cho tam giác $A B C$ nhọn có $H$ là trực tâm và $P$ là điểm di động bên trong tam giác $A B C$ sao cho $\angle B P C=\angle B H C$. Đường thẳng qua $B$ và vuông góc với $A B$ cắtPC tại $M$.Đường thẳng qua $C$ và vuông góc với $A C$ cắt $P B$ tại N. Chứng minh rằng trung điểm I của $M N$ luôn thuộc một đường cố định.

Hết

Giải

Bài 1.

Trong phương trình đã cho, thay $x=y=0$, ta có $f(f(0))=0$. \medskip

Lại thay $y=0$ thì $$f(f^3+f(0))=x^2f(x), \, \forall x.$$

Thay $y=f(0)$ thì $$f(x^3+f(0))=2f(0)+x^2f(x).$$

Từ đây suy ra $f(0)=0$. Thay $y=0$ vào đẳng thức đã cho ta được $f(x^3)=x^2f(x)$. Do đó ta có $$f(x^3+y+f(y))=2y+f(x^3) \text{ hay } f(x+y+f(y))=2y+f(x). \eqno{(*)}$$
Thay $y$ bởi $-y$, ta được $$f(x-y+f(-y))=-2y+f(x).$$
Với $x$ bất kì, ta lấy $2y=f(x)$ ta được $f(x-y+f(-y))=0$ suy ra $x-y+f(-y)=0$. Do đó, ta được $f(-x)=f(-y+f(-y))=-2y=-f(x).$
Từ đây suy ra
$$f(x+f(y)+f(f(y)))=2f(y)+f(x).$$
Trong $(*)$ thay $x=-y$ ta được $f(f(y))=2y+f(-y)=2y-f(y)$, kết hợp với đẳng thức trên, ta được $$f(x+2y)=2f(y)+f(x).$$ Đến đây cho $x=0$ ta được $f(2y)=2f(y)$ nên ta được $f(x+y)=f(x)+f(y)$, tức là $f(x)$ cộng tính.
Đến đây ta sẽ tính $f((x+1)^3+(x-1)^3)$ theo hai cách như sau

  • $f((x+1)^3+(x-1)^3)=f(2x^3+6x)=2x^2f(x)+6f(x).$
  • $f((x+1)^3+(x-1)^3)=(x+1)^2f(x+1)+(x-1)^2f(x-1)=(x+1)^2(f(x)+f(1))+(x-1)^2(f(x)-f(1))=2x^2f(x)+2f(x)+4xf(1).$

So sánh hai đẳng thức trên, ta được $f(x)=xf(1)=ax$ với mọi $x$. Thử lại ta được $a=1, a=-2$. \medskip

Vậy các hàm cần tìm là $f(x)=x, f(x)=-2x$.

Bài 2.

Ta có
$$\begin{aligned} u_{n+1}-2 & =(u_n-2)(u_{n-1}-1)^2 \\
& = (u_{n-2}-1)^2(u_{n-1}-1)^2(u_{n-2}-2) \\
&= (u_{n-1}-1)^2(u_{n-2}-1)^2 \cdots (u_2-1)^2(u_1-2). \end{aligned} $$

Do đó $$u_{2014}+2009= 2011 \left[ (u_{2013}-1)^2(u_{2012}-1)^2 \cdots (u_2-1)^2 +1 \right].$$

Gọi $B$ là biểu thức trong dấu ngoặc vuông thứ hai. Ta có bổ đề quen thuộc là nếu $a^2+b^2$ chia hết cho số nguyên tố $p=4k+3$ thì $a,b$ cùng chia hết cho $p.$ Từ đây suy ra số $B$ có dạng $a^2+1$ nên nó sẽ không có ước nguyên tố dạng $4k+3$. \medskip

Vậy $u_{2014}+9$ chỉ có một ước nguyên tố $p \equiv 3 \pmod{4}$ duy nhất là $2011$.

Bài 3. Trước hết, ta chứng minh bổ đề sau \medskip

Định lý Ramsey Với $s,t$ là các số nguyên dương, gọi $R(s,t)$ là số đỉnh ít nhất cần có của một graph để trong đó luôn tồn tại một tập độc lập $s$ đỉnh hoặc một graph con đầy đủ $t$ đỉnh. Khi đó
$$R(s,t)\le C_{s+t-2}^{s-1}. \eqno{(*)}$$

Chứng minh
Ta sẽ chứng minh rằng $$R(s,t)\le R(s-1,t)+R(s,t-1).$$
Để ý rằng với $s=1$ hoặc $t=1$ thì $R(s,t)=1$. Do đó, nếu chứng minh được đánh giá này thì chỉ cần dùng tính chất của tam giác Pascal để có $$R(s,t)\le C_{s+t-3}^{s-2}+C_{s+t-3}^{s-1}=C_{s+t-2}^{s-1}.$$
Đặt $n$ là vế phải của (*) và xét graph $G$ có $n$ đỉnh. Xét $v\in G$ thì

  • Nếu như có ít nhất $R(s,t-1)$ đỉnh kề với $v$. Khi đó, theo định nghĩa thì trong tập đỉnh đó, sẽ luôn có một tập độc lập $s$ đỉnh hoặc một graph con đầy đủ $t-1$ đỉnh, ghép thêm đỉnh $v$ vào thì thỏa mãn điều kiện của $R(s,t).$
  • Nếu như có ít nhất $R(s-1,t)$ đỉnh không kề với $v$. Tương tự trên, trong tập đỉnh đó, cũng sẽ có một một graph con đầy đủ $t$ đỉnh hoặc tập độc lập $s-1$ đỉnh, ghép thêm đỉnh $v$ vào thì thỏa mãn điều kiện của $R(s,t).$

    Từ đó, ta thấy graph $G$ này thỏa mãn điều kiện của $R(s,t)$ nên theo tính nhỏ nhất thì $R(s,t)\le n.$

Trở lại bài toán, \medskip

Xét graph đơn vô hướng $G=(V,E)$ đại diện cho hội nghị khoa học đã nêu, trong đó $V$ là tập hợp các đại biểu và hai đỉnh được nối nhau nếu hai đại biểu tương ứng quen nhau. Ta gọi $T$ là tập hợp đỉnh biểu diễn cho thành viên của ban tổ chức. \medskip

Khi đó một ủy ban gồm $5$ thành viên là đại diện nếu như đó là một graph đầy đủ, còn đó là thách thức nếu đó là graph không có cạnh. Ta gọi các graph con như thế là graph con “chuẩn”. \medskip

Trong các đỉnh $V\backslash T,$ ta xóa dần dần các graph con chuẩn đến khi không thực hiện được nữa. Ta gọi tập hợp còn lại là $S.$ Ta sẽ chứng minh rằng $S\cup T$ có thể phân hoạch thành các graph con chuẩn như trên. \medskip

Theo định lý Ramsey, rõ ràng $|S| \le C_{8}^{4}=70$. Xét một đỉnh $v \in S$ thì giả thiết, $v$ kề với cả $280$ đỉnh của $T$ nên ta chọn ra trong đó $4$ đỉnh để ghép với $v$ tạo thành một graph con “chuẩn”. Cứ như thế thực hiện cho đến hết các phần tử của $S$, còn lại bao nhiêu phần tử trong $T$ thì chia đều ra thành các graph con “chuẩn” là được. \medskip

Bài toán được giải quyết.

Bài 4.

Tam giác $PBM$ cân tại $P$ nên bằng biến đổi góc, ta có

$$\angle{PBM}=\angle{PMB} \Rightarrow 2\angle{ABC}-\angle{MBC}= \angle{ACB}+\angle{MBC}.$$

Do đó $\angle{ABC}=2\angle{MBC}$ nên $BM$ là tia phân giác của $\angle{ABC}.$ Theo tính chất đường phân giác thì
$$\frac{MC}{MA}=\frac{BC}{BA}=\frac{BC}{AC}.$$

Lại có $CN$ là phân giác ngoài của $\angle{ACB}$ nên ta cũng có
$\frac{NA}{NB}=\frac{CA}{CB}.$ Gọi $I$ là trung điểm của $BC$ thì $I$ là điểm cố định. \medskip

Xét tam giác $ABC$ với $I$ thuộc $BC$ , $M$ thuộc $AC$ và $N$ thuộc $AB$ với

$$\frac{IB}{IC} \cdot \frac{MC}{MA} \cdot \frac{NA}{NB}=1 \cdot \frac{BC}{AC} \cdot \frac{AC}{BC}=1$$

thì theo định lý Menelaus đảo, ta có $M , N , I$ thẳng hàng. \medskip

Vậy $MN$ luôn đi qua điểm $I$ cố định.

Bài 5. 

Rõ ràng có thể chọn giá trị các biến thích hợp để $P>0$ nên để tìm giá trị lớn nhất của $P$ thì ta chỉ xét các số ${{x}_{1}},{{x}_{2}},\ldots ,{{x}_{2014}}$ đều khác $0$ và số các số âm là chẵn. Không mất tính tổng quát, giả sử
${{x}_{1}}\ge {{x}_{2}}\ge \ldots \ge {{x}_{2m}}>0>{{x}_{2m+1}}\ge \ldots \ge {{x}_{2014}}.$
Đổi dấu các số ${{y}_{k}}=-{{x}_{k}}>0$ với $2m+1\le k\le 2014.$ Khi đó ta viết lại
$$\left\{ \begin{aligned}
& {{x}_{1}}+{{x}_{2}}+\cdots +{{x}_{2m}}={{y}_{1}}+{{y}_{2}}+\cdots +{{y}_{2n}}=A \\
& x_{1}^{2}+x_{2}^{2}+\cdots +x_{2m}^{2}+y_{1}^{2}+y_{2}^{2}+\cdots +y_{2n}^{2}=2014 \\
\end{aligned} \right.$$
trong đó $m+n=1007$ (ngoài ra $m,n>0$ vì các số đã cho không thể toàn bộ là dương hoặc toàn bộ là âm). Theo bất đẳng thức Cauchy-Schwarz thì
$$2014\ge \frac{{{A}^{2}}}{2m}+\frac{{{A}^{2}}}{2n} \text{ nên } {{A}^{2}}\le 4mn.$$
Lại theo bất đẳng thức AM-GM thì
$$\begin{aligned} P& =({{x}_{1}}{{x}_{2}}\ldots {{x}_{2m}})({{y}_{1}}{{y}_{2}}\ldots {{y}_{2n}})\le {{\left( \frac{A}{2m} \right)}^{2m}}{{\left( \frac{A}{2n} \right)}^{2n}} \\
&=\frac{{{A}^{2m+2n}}}{{{2}^{2m+2n}}{{m}^{2m}}{{n}^{2n}}}\le \frac{{{(4mn)}^{m+n}}}{{{2}^{2m+2n}}{{m}^{2m}}{{n}^{2n}}}={{\left( \frac{m}{n} \right)}^{n-m}}. \end{aligned}$$

Do $m,n$ khác tính chẵn lẻ nên với vai trò bình đẳng của $m,n,$ ta có thể giả sử $m<n$ nên $n-m\ge 1$ và $m\le 503.$ Khi đó, áp dụng bất đẳng thức Bernoulli thì

$${{\left( \frac{n}{m} \right)}^{n-m}}\ge 1+\left( \frac{n}{m}-1 \right)(n-m)=1+\frac{{{(n-m)}^{2}}}{m}\ge 1+\frac{1}{503}=\frac{504}{503}.$$
Suy ra $P\le {{\left( \frac{m}{n} \right)}^{n-m}}\le \frac{503}{504}.$ Đây chính là giá trị lớn nhất cần tìm, dấu bằng xảy ra khi
$m=503,n=504$ và $${{x}_{1}}={{x}_{2}}=\cdots ={{x}_{1006}}=\sqrt{\frac{504}{503}},{{x}_{1007}}={{x}_{1008}}=\cdots ={{x}_{2014}}=-\sqrt{\frac{503}{504}}.$$

Bài 6.

Xét hàm số $f(x)=\frac{x}{\sqrt{{{x}^{2}}+1}+\sqrt{2}}$ với $x\in \mathbb{R}$ thì $${f}'(x)=\frac{1+\sqrt{2+2{{x}^{2}}}}{\sqrt{1+{{x}^{2}}}{{\left( \sqrt{2}+\sqrt{1+{{x}^{2}}} \right)}^{2}}}>0$$ nên hàm này đồng biến trên $\mathbb{R}.$
Dãy số đã cho được viết lại thành
$$\left\{ \begin{aligned}
& {{u}_{1}}=1, \\
& {{u}_{n+1}}=f({{u}_{n}}),n\ge 1 \\
\end{aligned} \right.$$ thì ${{u}_{1}}<{{u}_{2}}$ nên dễ dàng chứng minh quy nạp được rằng dãy này giảm. \medskip

Do dãy này bị chặn dưới bởi $0$ nên nó có giới hạn, đặt giới hạn đó là $L\ge 0$. Trong biểu thức xác định dãy, cho $n\to +\infty ,$ ta được $$L=\frac{L}{\sqrt{{{L}^{2}}+1}+\sqrt{2}}$$ nên $L=0.$
Từ đó suy ra
$$\underset{n\to +\infty }{\mathop{\lim }}\,\frac{{{u}_{n+1}}}{{{u}_{n}}}=\underset{n\to +\infty }{\mathop{\lim }}\,\frac{1}{\sqrt{u_{n}^{2}+1}+\sqrt{2}}=\frac{1}{1+\sqrt{2}}.$$

Bài 7.

(a) Nếu $A=X$ thì $$S(A)=\sum\limits_{E\subset X}(-1)^{|E|}=C_n^0-C_n^1+C_n^2-\cdots +(-1)^nC_n^n=0.$$

Còn nếu $A\neq X$, do $S(A)$ chỉ phụ thuộc vào số phần tử của $A$ nên không mất tính tổng quát, ta giả sử rằng $A=\{1,2,\ldots ,k\}$ với $k<n$. Khi đó, ta có
$$\begin{aligned} S(A) & =\sum_{E\subset X-\{k\}}(-1)^{|E\cap A|}+\sum_{E\subset X-\{k\}}(-1)^{|(E\cup\{k\})\cap A|} \\
& =\sum_{E\subset X-\{k\}}(-1)^{|E\cap A|}-\sum_{E\subset X-\{k\}}(-1)^{|E\cap A|}=0. \end{aligned} $$
Vậy $S(A)=0,\forall A\subset X$. \medskip

(b) Đặt $f(E)=\sum_{i=1}^{m}(-1)^{|E\bigcap A_i|}a_i$. Giả sử $f(E)\leq 0, \, \forall E$. Mà ta cũng có
$$\sum_{E\subset X}f(E)=\sum_{i=1}^ma_iS(A_i)=0.$$
Suy ra $f(E)=0,\, \forall E \subset X$, nhưng điều này là không thể vì $f(\varnothing)<0$. Vậy luôn tồn tại $E$ sao cho $f(E)>0$.

 

Bài 8. 

Vẽ đường kính $AA’$ của đường tròn $(ABC)$. Vì $A’B \perp AB$ nên $B,A’,M$ thẳng hàng. Tương tự thì $C,A’,N$ thẳng hàng. Giả sử
$BA’, CA’$ cắt lại $(BHC)$ lần lượt tại $E,F$. Mặt khác

$$\angle NPM=180{}^\circ -\angle BHC=\angle A=180{}^\circ -\angle B{A}’C=\angle M{A}’N$$

nên $PA’MN$ là tứ giác nội tiếp.

Ta sẽ chứng minh trung điểm của $A’F, A’E, MN$ là thẳng hàng. Theo định lý Menelaus đảo thì điều nào tương đương với $$ \dfrac{\overline{A’F}}{\overline{A’N}} = \dfrac{\overline{EA’}}{\overline{EM}} \Leftrightarrow \dfrac{\overline{A’F}}{\overline{A’E}} = – \dfrac{\overline{A’N}}{\overline{EM}} \Leftrightarrow \dfrac{A’B}{A’C} = \dfrac{A’N}{ME}. \eqno{(*)}$$

Vì $\angle BN{A}’=\angle CME$ và $\angle NB{A}’=\angle MCE$ nên hai tam giác $BN{A}’,CME$ đồng dạng với nhau. Do đó
$\frac{{A}’N}{ME}=\frac{{A}’B}{CE}$.
Mặt khác, bằng biến đổi góc, ta cũng có $C{A}’E$ cân tại $C$ nên $CE=C{A}’.$ Ta có được $$\frac{{A}’N}{ME}=\frac{{A}’B}{{A}’C}.$$
Do đó, khẳng định $(*)$ là đúng. Vậy nên điểm $I$ luôn nằm trên đường trung bình của tam giác $A’EF$ là đường cố định.

Bạn đọc có thể tìm thêm nhiều cách giải cho bài 8 này tại

link sau

Tham khảo từ sách “Tuyển tập đề thi môn Toán đội tuyển và dự tuyển trường PTNK”

Đề thi và đáp án tuyển sinh vào lớp 10 PTNK không chuyên 2014

Đề thi vào lớp 10 trường Phổ thông Năng khiếu năm 2014

Bài 1.

a) Giải phương trình $\left( 3-x \right) \sqrt{\left( 3+x \right) \left( 9+x^2 \right) } = 4 \sqrt{5 \left( 3-x \right) }$

b) Tính $\dfrac{x}{y}$ biết $x>1$, $y<0$ và $ \dfrac{\left( x+y \right) \left( x^3 -y^3 \right) \sqrt{\left( 1- \sqrt{4x-1} \right)^2 }}{\left( 1-\sqrt{4x-1} \right) \left( x^2y^2 + xy^3 +y^4 \right) }=-6$.

Giải

a) $(3-x) \sqrt{(3+x) \left( 9+x^2 \right) }=4\sqrt{5(3-x)}$

Điều kiện $\left\{ \begin{array}{l} 3-x \ge 0 \\ (x+3)\left( x^2 +9 \right) \ge 0 \end{array}\right. \Leftrightarrow -3\le x \le 3$

Với điều kiện trên ta có:

$(3-x) \sqrt{(3+x) \left( 9+x^2 \right) }=4\sqrt{5(3-x)} $

$\Leftrightarrow \sqrt{3-x}\left( \sqrt{3-x}\sqrt{(3+x)\left( x^2+9 \right) }-4\sqrt{5} \right) =0 $

$\Leftrightarrow \sqrt{3-x}\left( \sqrt{81-x^4} – 4\sqrt{5} \right) =0 $

$\Leftrightarrow \left[ \begin{array}{l} \sqrt{3-x}=0 \\ \sqrt{81-x^4}=4\sqrt{5} \end{array}\right. \Leftrightarrow \left[ \begin{array}{l} x=3 \\ x^4=1 \end{array}\right. \Leftrightarrow \left[ \begin{array}{l} x=3\\ x=-1 \\ x=1 \end{array}\right.$

Vậy $S=\left\{ 3; -1;1 \right\} $

b) Ta có $x>1 \Rightarrow \sqrt{4x-1}-1>0 \Rightarrow \sqrt{\left( 1-\sqrt{4x-1} \right) ^2 }=\sqrt{4x-1} -1 $

Do đó:

$\dfrac{(x+y)\left( x^3-y^3 \right) \sqrt{\left( 1-\sqrt{4x-1} \right) ^2}}{\left( 1-\sqrt{4x-1} \right) \left( x^2y^2+xy^3+y^4 \right) } =-6 $

$\Leftrightarrow \dfrac{(x+y)(x-y)\left( x^2 + xy+y^2 \right) }{y^2\left( x^2+xy+y^2 \right) } =6 $

$\Leftrightarrow x^2-y^2=6y^2 \Leftrightarrow \dfrac{x^2}{y^2}=7 \Rightarrow \dfrac{x}{y}=-\sqrt{7}$ (do $x>1$, $y<0$)

Bài 2.

a) Giải hệ phương trình $\left\{ \begin{array}{l} \left( x^2 -y +2 \right) \left( \sqrt{\left( x^2 +9 \right) \left( y+7 \right)} -15 \right) =0\\ \sqrt{x^2 + 9} + \sqrt{y+7} =8 \end{array}\right. $

b) Hình thoi $ABCD$ có diện tích là $18\sqrt{3}$ (mét vuông), tam giác $ABD$ đều. Tính chu vi hình thoi và bán kính đường tròn ngoại tiếp tam giác $ABC$.

Giải

a) $\left\{ \begin{array}{l} \left( x^2-y+2 \right) \left( \sqrt{\left( x^2 +9 \right) \left( y+7 \right)}-15 \right) =0 \ (1)\\ \sqrt{x^2+9}+\sqrt{y+7}=8 \ (2) \end{array}\right. $ (điều kiện $y\ge -7$)

$(1) \Leftrightarrow \left[ \begin{array}{l} x^2=y-2 \\ \sqrt{\left( x^2+9 \right) \left( y+7 \right) } =15 \end{array}\right. $

Với $x^2=y-2$ thế vào $(2)$ ta có: $2\sqrt{y+7}=8\Leftrightarrow y=9 \Rightarrow x= \pm \sqrt{7}$

Ta có nghiệm $(x;y)$ là $\left( \sqrt{7};9 \right) $, $\left( -\sqrt{7};9 \right) $

Với $\sqrt{\left( x^2 +9 \right) \left( y+7 \right) }=15 $, đặt $u= \sqrt{x^2+9}$, $v=\sqrt{y+7}$ ($u,v \ge 0$) ta có hệ

$\left\{ \begin{array}{l} uv=15 \\ u+v=8 \end{array}\right.$ $ \Leftrightarrow \left\{ \begin{array}{l} u=3 \\ v=5 \end{array}\right.$ hoặc $\left\{ \begin{array}{l} u=5 \\ v=3 \end{array} \right. $

Với $u=3$, ta có $x=0$, $v = 5$ ta có $y = 18$. Ta có nghiệm $(0;18)$

Với $u = 5$, ta có $x = 4$ hoặc $x = – 4$, $v = 3$ ta có $y = 2$.

Vậy hệ phương trình có $5$ nghiệm $\left( \sqrt{7};9 \right) $, $\left( -\sqrt{7};9 \right) $, $(0;18)$, $(4;2)$, $(-4;2)$.

b) Gọi $O$ là giao điểm của $AC$ và $BD$. Gọi $a$ là cạnh hình thoi. Tam giác $ABD$ đều nên $BD = AB = a$, $\angle{ABD}=60^\circ $.

$AO=AB \sin \angle{ABD} =AB \sin 60^\circ  = \dfrac{a\sqrt{3}}{2} \Rightarrow AC=2AO=a\sqrt{3}$.

Ta có $S_{ABCD} =\dfrac{1}{2} AC.BD=18\sqrt{3} \Leftrightarrow \dfrac{1}{2}a\sqrt{3}\cdot a=18\sqrt{3}\Leftrightarrow a=6$ $(m)$, khi đó chu vi hình thoi là $4a=24$ $(m)$.

Hơn nữa $DA = DB = DC = a$ nên $D$ là tâm đường tròn ngoại tiếp tam giác $ABC$ và bán kính bằng $6m$.

Bài 3. Cho phương trình $\dfrac{mx^2 + (m-3)x +2m-1}{x+3}=0$ $(1)$

a) Giải phương trình $(1)$ khi $m=-1$.

b) Tìm m để phương trình $(1)$ có $2$ nghiệm phân biệt $x_1$, $x_2$ sao cho

$$21x_1 + 7m \left( 2+ x_2 + x_2^2 \right) = 58 $$

Giải

a) Khi m=-1 ta có phương trình:

$\dfrac{-x^2 -4x-3}{x+3}=0 \,\, (\text{dk: } x \ne -3) $ $\Leftrightarrow -x^2 -4x-3 =0 \Leftrightarrow \left[ \begin{array}{l} x=-1 \,\,(n) \\ x=-3 \,\, (l) \end{array}\right. $

Vậy $S=\left\{ -1 \right\} $

b) $\dfrac{mx^2+(m-3)x+2m-1}{x+3}=0$ $(1)$

Điều kiện để phương trình có hai nghiệm phân biệt $x_1,x_2$ là phương trình $mx^2+(m-3)x+2m-1=0$ có hai nghiệm phân biệt khác $-3$

$\left\{ \begin{array}{l} m \ne 0 \\ \Delta = (m-3)^2 -4m(2m-1) >0 \\ m(-3)^2+(m-3)(-3)+2m-1 \ne 0 \end{array}\right. $

$\Leftrightarrow \left\{ \begin{array}{l} m \ne 0\\ 7m^2 +2m-9 <0 \\ m \ne -1 \end{array}\right.$

$\Leftrightarrow \left\{ \begin{array}{l} m\ne 0\\ m \ne -1 \\ -\dfrac{9}{7} < m < 1 \end{array}\right. $

Ta có $mx_2^2 + (m-3) x_2 +2m-1 =0 \Leftrightarrow m \left( 2+x_2 + x_2^2 \right) =3x_2 +1$

Do đó  $21x_1 + 7m \left( 2+ x_2 + x_2^2 \right) =58 \Leftrightarrow 21x_1 + 7(3x_2 +1 ) =58 $

$\Leftrightarrow 21 \left( x_1 +x_2 \right) =51 \Leftrightarrow x_1 + x_2 =\dfrac{17}{7} $

$\Leftrightarrow \dfrac{3-m}{m} = \dfrac{17}{7} \Leftrightarrow 21-7m =17m \Leftrightarrow m=\dfrac{7}{8} \,\, (n) $ $

Vậy $m=\dfrac{7}{8}$

Bài 4.

a) Gọi $x= \dfrac{a+b}{2}$, $y=\sqrt{ab}$ lần lượt là trung bình cộng và trung bình nhân của hai số dương $a, b$. Biết trung bình cộng của $x$ và $y$ bằng $100$. Tính $S = \sqrt{a}+\sqrt{b}$

b) Giả sử hai đại lượng $x, y$ tỉ lệ nghịch ($x, y$ luôn dương). Nếu $x$ tăng $a \% $ thì $y$ giảm $m \%$. Tính $m$ theo $a$.

Giải

a) Ta có $100=\dfrac{x+y}{2}=\dfrac{\dfrac{a+b}{2}+\sqrt{ab}}{2} = \dfrac{a+b+2\sqrt{ab}}{4} = \dfrac{\left( \sqrt{a}+\sqrt{b} \right)^2}{4} $

$\Leftrightarrow \left( \sqrt{a}+\sqrt{b} \right) ^2 =400 \Leftrightarrow \sqrt{a} +\sqrt{b}=20$

b) Khi $x$ tăng $a\% $ thì được $\left( 1+ \dfrac{a}{100} \right) x$, y giảm $m\% $ thì được $\left( 1- \dfrac{m}{100} \right) y$.

Do $x$, $y$ tỷ lệ nghịch nên ta có phương trình:

$xy= \left( 1+ \dfrac{a}{100} \right) x \left( 1- \dfrac{m}{100} \right) y $

$\Leftrightarrow 10000 = (100+a) (100-m)$

$\Leftrightarrow m= \dfrac{100a}{100+a}$

Bài 5. Cho hình vuông $ABCD$ có $AB=2a$, $AC$ cắt $BD$ tại $I$. Gọi $T$ là đường tròn ngoại tiếp tam giác $CID$, $BE$ tiếp xúc với $T$ tại $E$ ($E$ khác $C$). $DE$ cắt $AB$ tại $F$.

a) Chứng minh tam giác $ABE$ cân. Tính $AF$ theo $a$.

b) $BE$ cắt $AD$ tại $P$. Chứng minh đường tròn ngoại tiếp tam giác $ABP$ tiếp xúc với $CD$. Tính $\dfrac{AP}{PD}$

c) $AE$ cắt $T$ tại $M$ ($M$ khác $E$). Tính $AM$ theo $a$.

Giải

a) Gọi $T$ là trung điểm của $CD$, tam giác $CID$ vuông cân tại $I$ nên $T$ là tâm đường tròn ngoại tiếp tam giác $CID$.

Ta có $BE$ và $BC$ là hai tiếp tuyến của $T$ nên $BE = BC$, mà $BC = BA$ nên $BE = BA$ hay tam giác $ABE$ cân tại $B$.

Ta có $\angle{DEC}=90^0$, suy ra $DF \bot CE$ mà $CE \bot BT$ (t/c hai tiếp tuyến cắt nhau), suy ra $DF //BT$ mà $BF // DT $ nên $BFDT$ là hình bình hành, suy ra $BF = DT = a$. Suy ra $AF = a$

b) Ta có $PE$, $PD$ là tiếp tuyến của $(T)$ nên $PD = PE$.

Khi đó $BP = EB + EP = AB+PD=BC+PD$.

Gọi $K$ là trung điểm của $BP$, tam giác $APB$ vuông nên $K$ là tâm đường tròn ngoại tiếp tam giác $ABP$ và bán kính đường tròn bằng $\dfrac{1}{2} PB$.

Tứ giác $DPBC$ là hình thang vuông có $KT$ là đường trung bình, suy ra $KT = \dfrac{1}{2} (DP + BC) = \dfrac{1}{2} PB$ và $KT//PD$, suy ra $KT \bot CD$.

Do đó khoảng cách từ $K$ đến $CD$ bằng bán kính của $(K)$ nên $CD$ tiếp xúc với đường tròn ngoại tiếp tam giác $APB$.

Ta có $TP$ và $TB$ là phân giác của $\angle{ETD}$ và $\angle{ETC}$ nên $\angle{BTP}$ vuông. Khi đó $EP. EB=TE^2$, suy ra $EP = \dfrac{TE^2}{BE} =\dfrac{a^2}{2a}=\dfrac{1}{2}a$

Khi đó $PD = PE =\dfrac{1}{2}a$, suy ra $PA =\dfrac{3}{2}a$. Suy ra $\dfrac{AP}{DP}=3$

c) Tứ giác $AEIF$ có $\angle{IEF}=\angle{DCI}=45^\circ =\angle{IAF}$ suy ra tứ giác $AEIF$ nội tiếp

Do đó $\angle{IEA}=\angle{IFA}=90^\circ $ và $EM$ là phân giác $\angle{CED}$

Khi đó $IM$ là đường kính và $M$ là điểm chính giữa cung $CD$ của $T$

Suy ra $\angle{ICM}=90^0$, $CM=CI=a\sqrt{2}$.

Khi đó $AM^2 = AC^2 + CM^2 = 8a^2 +2a^2 =10a^2 \Rightarrow AM = a\sqrt{10}$.

 

Đề thi và đáp án thi vào lớp 10 TPHCM 2014

I. ĐỀ thi vào lớp 10 TPHCM 2014

Bài 1. Giải các phương trình và hệ phương trình sau:
a) $x^2-7x+12 = 0$
b) $x^2-(\sqrt{2}+1)x+\sqrt{2} = 0$
c) $x^4-9x^2+20=0$
d) $3x-2y=4$ và $ 4x-3y=5. $

Bài 2.
a) Vẽ đồ thị (P) của hàm số $y = x^2$ và đường thẳng $(D):y=2x+3$ trên cùng một hệ trục tọa độ.
b) Tìm tọa độ giao điểm của (P) và (D) ở câu trên bằng phép tính.
Bài 3. Thu gọn các biểu thức sau:
a) $A = \frac{{5 + \sqrt 5 }}{{\sqrt 5 + 2}} + \frac{{\sqrt 5 }}{{\sqrt 5 – 1}} – \frac{{3\sqrt 5 }}{{3 + \sqrt 5 }}$
b) $B = \left( {\frac{x}{{x + 3\sqrt x }} + \frac{1}{{\sqrt x + 3}}} \right):\left( {1 – \frac{2}{{\sqrt x }} + \frac{6}{{x + 3\sqrt x }}} \right)$ với $x > 0$.
Bài 4. Cho phương trình $x^2-mx-1=0$ (1) ($x$ là ẩn).

a) Chứng minh phương trình (1) luôn có hai nghiệm trái dấu.
b) Gọi $x_1, x_2$ là các nghiệm của phương trình (1). Tính giá trị của biểu thức $P = \dfrac{x_1^2+x_1-1}{x_1} – \dfrac{x_2^2+x_2-1}{x_2}$.
Bài 5. Cho tam giác $ABC$ có ba góc nhọn, nội tiếp đường tròn tâm $O$ $(AB < AC)$. Các đường cao $AD$ và $CF$ của tam giác $ABC$ cắt nhau tại $H$.
a) Chứng minh tứ giác $BFHD$ nội tiếp. Suy ra $\angle AHC = 180^o – \angle ABC$.
b) Gọi $M$ là điểm bất kì trên cung nhỏ $BC$ của đường tròn $(O)$. ($M$ khác $B$ và $C$) và $N$ là điểm đối xứng của $M$ qua $AC$. Chứng minh tứ giác $AHCN$ nội tiếp.
c) Gọi $I$ là giao điểm của $AM$ và $HC$. $J$ là giao điểm của $AC$ và $HN$. Chứng minh $\angle AJI = \angle ANC$.
d) Chứng minh rằng $OA$ vuông góc với $IJ$.

II. ĐÁP ÁN

Bài 1.
a) $x^2 – 7x +12 =0$
$\Delta =1 $
Hai nghiệm của phương trình là $x_1 = 3$; $x_2 =4$
b)  $x^2 – \left( \sqrt{2}+1 \right) + \sqrt{2} = 0 $
Phương trình có $a+b+c = 0$ nên hai nghiệm là $x_1=1$; $x_2 = \sqrt{2}$
c)  $x^4 – 9x^2 +20 =0$
Đặt $t= x^2 \ge 0$
Phương trình trở thành: $t^2 -9t +20 =0$
$\Delta =1 $
$t_1 =4$ (nhận) và $t_2 =5$ (nhận)
Với $t=4 \Rightarrow x= \pm 2$; với $t=5 \Rightarrow x= \pm \sqrt{5}$
d)  $3x-2y=4  (1) $ và $4x-3y =5  (2)$
$\Leftrightarrow  3x-2y=4  (1) $ và  $x= 2  (3\cdot (1) – 2 \cdot (2))$
$\Leftrightarrow  x=2$ và $y=1$.

Bài 2.
a) Đồ thị:

Lưu ý: $(P)$ đi qua $O(0;0)$, $( \pm 1 ;1)$, $\pm 2; 4 )$
$(D)$ đi qua $(-1;1)$, $(0;3)$
b) Phương trình hoành độ giao điểm của $(P)$ và $(D)$ là:
$x^2 = 2x+3 \Leftrightarrow x^2 -2x -3 =0$

$\Leftrightarrow  x = -1$ và $x= 3$
$y(-1) = 1$; $y(3) =9 $
Vậy tọa độ giao điểm của $(P)$ và $(D)$ là $(-1;1)$, $(3;9)$.
Bài 3.
a) $A= \dfrac{5+ \sqrt{5}}{\sqrt{5}+2} + \dfrac{\sqrt{5}}{\sqrt{5}-1}- \dfrac{3\sqrt{5}}{3+ \sqrt{5}} $
$= \dfrac{\left( 5+ \sqrt{5} \right) \left( \sqrt{5}-2 \right) }{1} + \dfrac{\sqrt{5}\left( \sqrt{5}+1 \right) }{4} – \dfrac{3\sqrt{5}\left( 3- \sqrt{5} \right) }{4} $
$= 3\sqrt{5}-5 + \dfrac{5+ \sqrt{5}-9\sqrt{5}+15}{4} $
$=3\sqrt{5}-5 + 5 -2\sqrt{5} = \sqrt{5}$.
b) $B=\left( \dfrac{x}{x+ 3\sqrt{x}}+ \dfrac{1}{\sqrt{x}+3} \right) : \left( 1- \dfrac{2}{\sqrt{x}} + \dfrac{6}{x+ 3\sqrt{x}} \right) \hspace{1.5cm} (x > 0) $
$= \left( \dfrac{\sqrt{x}}{\sqrt{x}+3} + \dfrac{1}{\sqrt{x}+3} \right) : \left( \dfrac{x+ 3\sqrt{x}- 2 \left( \sqrt{x} + 3 \right) + 6}{\sqrt{x} \left( \sqrt{x}+ 3 \right) } \right) $
$= \left( \dfrac{\sqrt{x}+ 1}{\sqrt{x} + 3} \right) : \left( \dfrac{\sqrt{x}+ 1}{\sqrt{x}+3} \right) =1$
Bài 4.

a) $x^2 – mx -1 =0$ $(1)$
$\Delta = m^2 + 4 >0$
Do đó phương trình luôn có hai nghiệm phân biệt với mọi $m$.
Theo Viet, ta có: $P = x_1 \cdot x_2 = \dfrac{c}{a} = -1 <0 $
Vậy phương trình luôn có hai nghiệm trái dấu.
b) Theo Viet, ta có:

$S= x_1 + x_2 = m $ và  $P = x_1 \cdot x_2 = -1$
$P = \dfrac{x_1^2 + x_1 -1}{x_1} – \dfrac{x_2^2 + x_2 -1}{x_2} $
$= \dfrac{x_1^2 + x_1 + x_1 x_2}{x_1} – \dfrac{x_2^2 + x_2 + x_1 x_2 }{x_2} $
$= x_1 + 1 + x_2 – x_2 -1 -x_1 =0$
Bài 5.


a) Ta có:
$\angle BFC = \angle BDA = 90^ \circ$ ($AD$, $CF$ là các đường cao)
$\Rightarrow \angle BFC + \angle BDA =180^ \circ \Rightarrow $ tứ giác $BFHD$ nội tiếp
$\Rightarrow \angle ABC + \angle DHF =180 ^\circ $
$\angle ABC + \angle AHC = 180 ^\circ $
$\angle AHC = 180 ^\circ – \angle ABC$.
b) Ta có $\angle AMC = \angle ABC$ ( cùng chắn cung $AC$)
$\angle AMC = \angle ANC$ (tính chất đối xứng)
$\Rightarrow \angle ANC = \angle ABC$
Mà $\angle AHC + \angle ABC = 180 ^\circ$
$\Rightarrow \angle AHC + \angle ANC = 180 ^\circ$
$\Rightarrow $ $AHCN$ nội tiếp.
c) Ta có $\angle MAC = \angle NAC$ ( tính chất đối xứng)
$\angle NAC = \angle NHC $ (cùng chắn cung $NC$)
$\Rightarrow \angle MAC = \angle NHC$ hay $\angle IAJ = \angle IHJ $
$\Rightarrow $ $AHIJ$ nội tiếp (2 đỉnh kề cùng nhìn cạnh dưới góc bằng nhau)
$\Rightarrow \angle AJI = 180 ^\circ \angle AHC = \angle ANC$.
d) Vẽ tiếp tuyến $xy$ của $(O)$ tại $A$ $\Rightarrow OA \bot xy$
$\angle AJI = \angle ANC = \angle AMC = \angle yAC \Rightarrow IJ // xy $
$\Rightarrow OA \bot IJ$.

Đáp án toán PTNK 2014

Bài 1. (Toán chung) Cho hình vuông $ABCD$ có $AB=2a$, $AC$ cắt $BD$ tại $I$. Gọi $T$ là đường tròn ngoại tiếp tam giác $CID$, $BE$ tiếp xúc với $T$ tại $E$ ($E$ khác $C$). $DE$ cắt $AB$ tại $F$.
a. Chứng minh tam giác $ABE$ cân. Tính $AF$ theo $a$.
b. $BE$ cắt $AD$ tại $P$. Chứng minh đường tròn ngoại tiếp tam giác $ABP$ tiếp xúc với
$CD$. Tính $\dfrac{AP}{PD}$
c. $AE$ cắt $T$ tại $M$ ($M$ khác $E$). Tính $AM$ theo $a$.

Gợi ý

a.

  • Gọi $T$ là trung điểm của $CD$, tam giác $CID$ vuông cân tại $I$ nên $T$ là tâm đường tròn ngoại tiếp tam giác $CID$.
  • Ta có $BE$ và $BC$ là hai tiếp tuyến của $T$ nên $BE = BC$, mà $BC = BA$ nên $BE = BA$ hay tam giác $ABE$ cân tại $B$.
  • Ta có $\angle{DEC}=90^0$, suy ra $DF \bot CE$ mà $CE \bot BT$ (t/c hai tiếp tuyến cắt nhau), suy ra $DF //BT$ mà $BF // DT $ nên $BFDT$ là hình bình hành, suy ra $BF = DT = a$. Suy ra $AF = a$

b.

  • Ta có $PE$, $PD$ là tiếp tuyến của $(T)$ nên $PD = PE$. Khi đó $BP = EB + EP = AB+PD=BC+PD$.
  • Gọi $K$ là trung điểm của $BP$, tam giác $APB$ vuông nên $K$ là tâm đường tròn ngoại tiếp tam giác $ABP$ và bán kính đường tròn bằng $\dfrac{1}{2} PB$.
  • Tứ giác $DPBC$ là hình thang vuông có $KT$ là đường trung bình, suy ra $KT = \dfrac{1}{2} (DP + BC) = \dfrac{1}{2} PB$ và $KT//PD$, suy ra $KT \bot CD$.
  • Do đó khoảng cách từ $K$ đến $CD$ bằng bán kính của $(K)$ nên $CD$ tiếp xúc với đường tròn ngoại tiếp tam giác $APB$.
  • Ta có $TP$ và $TB$ là phân giác của $\angle{ETD}$ và $\angle{ETC}$ nên $\angle{BTP}$ vuông. Khi đó $EP. EB=TE^2$, suy ra $EP = \dfrac{TE^2}{BE} =\dfrac{a^2}{2a}=\dfrac{1}{2}a$ Khi đó $PD = PE =\dfrac{1}{2}a$, suy ra $PA =\dfrac{3}{2}a$. Suy ra $\dfrac{AP}{DP}=3$

c.

  • Tứ giác $AEIF$ có $\angle{IEF}=\angle{DCI}=45^0=\angle{IAF}$, suy ra tứ giác $AEIF$ nội tiếp, do đó $\angle{IEA}=\angle{IFA}=90^0$ và $EM$ là phân giác $\angle{CED}$. Khi đó $IM$ là đường kính và $M$ là điểm chính giữa cung $CD$ của $T$. Suy ra $\angle{ICM}=90^0$, $CM=CI=a\sqrt{2}$.
  • Khi đó $AM^2 = AC^2 + CM^2 = 8a^2 +2a^2 =10a^2 \Rightarrow AM = a\sqrt{10}$.

Bài 2. (Toán chuyên) Cho tam giác $ABC$ vuông tại $A$ với các đường phân giác trong $BM, CN$. Chứng minh bất đẳng thức $\dfrac{(MC+MA)(NB+NA)}{MA.NA} \geq 3 + 2\sqrt{2}$.

Gợi ý
  • Áp dụng tính chất đường phân giác ta có:
    $\dfrac{MC}{MA} = \dfrac{BC}{AB}$, suy ra $\dfrac{MC+MA}{MA} = 1 + \dfrac{BC}{AB}$.
  • $\dfrac{NB}{NA} = \dfrac{BC}{AC}$, suy ra $\dfrac{BN+NA}{NA} = 1+ \dfrac{BC}{AC}$.
  • Suy ra:\\ $\dfrac{(MC+MA)(NB+NA)}{MA.NA} = \left(1+\dfrac{BC}{AB}\right)\left(1+\dfrac{BC}{AC}\right) = 1 + \dfrac{BC^2}{AB.AC}+ \dfrac{BC}{AB}+ \dfrac{BC}{AC}$.
  • Ta có $BC^2 = AB^2 + AC^2 \geq 2.AB.AC$, suy ra $\dfrac{BCC^2}{AB.AC} \geq 2$.
  • Và $\dfrac{BA}{AC} +\dfrac{BC}{AC} \geq \sqrt{\dfrac{BC.BC}{AB.AC}} \geq 2\sqrt{2}$.
  • Do đó $\dfrac{(MC+MA)(NB+NA)}{MA.NA} \geq 3 + 2\sqrt{2}$.

Bài 3. (Toán chuyên) Cho điểm C thay đổi trên nửa đường tròn đường kính $AB = 2R$ ($C \neq A, C \neq B$). Gọi $H$ là hình chiếu vuông góc của $C$ lên $AB$; $I$ và $J$ lần lượt là tâm đường tròn nội tiếp các
tam giác $ACH$ và $BCH$. Các đường thẳng $CI, CJ$ cắt $AB$ tại $M, N$.
a. Chứng minh $AN = AC, BM = BC$.
b. Chứng minh 4 điểm $M, N, I, J$ cùng nằm trên một đường tròn và các đường thẳng
$MJ, NI$ và $CH$ đồng quy.
c. Tìm giá trị lớn nhất của MN và giá trị lớn nhất của diện tích tam giác $CMN$ theo $R$.

Gợi ý

a.

  • Ta có $\angle HCB = \angle CAB$ (cùng phụ với $\angle ABC$) và $\angle HCA = \angle CBA$ (cùng phụ với $\angle BAC)$.
  • Ta có $\angle CAN =\angle NAC + \angle ABC = \angle HAN + \angle ACB = \angle CAN$. Suy ra tam giác $CAN$ cân tại $A$ hay $AN = AC$. Chứng minh tương tự ta có $BM = BC$.

b.

  • Tam giác $CAN$ cân tại $A$ có $AI$ là phân giác nên cũng là trung trực, suy ra $IC = IN$, suy ra $\angle INC = \angle ICN = \angle ICH + \angle NCH = \dfrac{1}{2} \angle ACH + \dfrac{1}{2} \angle BCH = 45^o$.
  • Tương tự thì $\angle JMC = 45^o$.
  • Tứ giác $MIJN$ có $\angle JMC = \angle INC = 45^o$ nên là tứ giác nội tiếp, hay $M, N, I, J$ cùng thuộc một đường tròn.
  • Tam giác $INC$ cân có $\angle ICN = 45^o$ nên $\angle CIN = 90^o$, suy ra $CI \bot CM$.
  • Chứng minh tương tự $MJ \bot CN$.
  • Tam giác $CMN$ có $CH, MJ, NI$ là các đường cao nên đồng quy.

c.

  • Đặt $AC = b, BC = a$. Ta có $a^2 + b^2 = BC^2 = 4R^2$.
  • Ta có $AN = AC = b, BM = BC = a$. \\$AM + BN = BC + MN$, suy ra $MN = a+b-BC = a+b-2R$.
  • Ta có $(a+b)^2 \leq 2(a^2+b^2) = 8R^2$. Suy ra $a+b \leq 2 \sqrt{2}R$, suy ra $a+b-2R \leq 2R(\sqrt{2}-1)$.
  • Đẳng thức xảy ra khi $a=b=R\sqrt{2}$.
  • Vậy giá trị lớn nhất của $MN$ bằng $2R(\sqrt{2}-1)$ khi $C$ là điểm chính giữa đường tròn.
    Khi đó $S_{CMN} = \dfrac{1}{2}CH.MN \leq R^2(\sqrt{2}-1)$.
  • Đẳng thức xảy ra khi $C$ là điểm chính giữa đường tròn.

Đường thẳng qua điểm cố định. VMO 2014.

Bài toán. (PoP1.12) (VMO 2014) Cho tam giác nhọn $ABC$ nội tiếp đường tròn $(O)$, trong đó $B, C$ cố định và $A$ thay đổi trên $(O)$. Trên các tia $AB$ và $AC$ lần lượt lấy các điểm $M$ và $N$ sao cho $MA = MC$ và $NA = NB$. Các đường tròn ngoại tiếp các tam giác $AMN$ và $ABC$ cắt nhau tại $P$ ($P \neq A$). Đường thẳng $MN$ cắt đường thẳng $BC$ tại $Q$.

  1. Chứng minh rằng ba điểm $A, P, Q$ thẳng hàng.
  2. Gọi $D$ là trung điểm của $BC$. Các đường tròn có tâm là $M, N$ và cùng đi qua $A$ cắt nhau tại $K$ ($K \neq A$). Đường thẳng qua $A$ vuông góc với $AK$ cắt $BC$ tại $E$. Đường tròn ngoại tiếp tam giác $ADE$ cắt $(O)$ tại $F (F \neq A)$. Chứng minh rằng đường thẳng $AF$ đi qua một điểm cố định.
Gợi ý

1.

  • Ta có $MA = MC$ và $NA = NB$ nên tam giác $MAC$ cân tại $M$ và tam giác $NAB$ cân tại $N$.
  • Do đó $\angle BMC = \angle BAC + \angle MAC = 2\angle BAC = \angle BOC$ hay tứ giác $BMOC$ nội tiếp.
  • Tương tự thì tứ giác $BONC$ nội tiếp nên $BMNC$ nội tiếp.
  • Khi đó $QM.QN = QB.QC$, lại có $APMN, APBC$ nội tiếp nên $A, P, Q$ thẳng hàng.

2.

  • Tam giác $AMN$ có $OM \bot AN, ON \bot AM$ nên $AO \bot MN$. Mặt khác $AK \bot MN$ nên $A, O, K$ thẳng hàng.
  • Ta có $\angle OAE = \angle ODE = 90^o$ nên $AODE$ nội tiếp, do đó $\angle OAE = \angle OFE = 90^o$. Hơn nữa $OA = OF$ nên $A, F$ đối xứng qua $OE$.
  • Giả sử $OE$ cắt $AF$ tại $H$ thì $EH.EO = EA^2= EB.EC$ nên $BHOC$ nội tiếp, lại có $\angle OHA = 90^o$ nên $AH$ đi qua $G$ là điểm chính giữa cung $BC$ không chứa $O$ của đường tròn ngoại tiếp tam giác $OBC$.
  • Vậy $AF$ luôn đi qua điểm $G$ cố định.